You are on page 1of 57

English Medium

VISION IAS
Test -23 TEST BOOKLET
GENERAL STUDIES (P) 2021 – Test – 3200
Test Booklet Series

C
Time Allowed: Two Hours Maximum Marks: 200

Basic Science and Technology


Basic Science and Technology Physics
Physics Standards and units
Standards and units Mechanics and properties of matter
Mechanics and properties of matter Heat
Heat Sound
Sound Optics
Optics Electricity and Magnetism
Electricity and Magnetism Atomic and Nuclear physics
Atomic and Nuclear physics Modern Physics
Modern Physics
Astronomy and Space Science Chemistry
Chemistry Matters and
Matters and its nature Chemical Reactions and Equations
Chemical Reactions and Equations Structure of the Atom
+ Applied Science and Modern Day Periodic classification of elements
Technology / Chemical Bonding

+ Current Affairs (Dec2020-JAN-2020


1. With reference to the Genetically Modified 4. The term 'Nanomicelles' recently seen in the
Golden rice, consider the following news is related to
statements: (a) Microscopic organisms recently carried
1. It is produced by using the genes from by Israel Space Agency to moon
the mustard plant and soil bacterium. (b) Nano structures used in the treatment of
2. It is a rich source of the Vitamin A. disease
3. Indian government allowed its (c) Microscopic organisms discovered in
distribution in selective aspirational Mariana trench by China
districts recently. (d) Microscopic structures used in the
Which of the statements given above is/are
manufacture of Graphite sheets
correct?
(a) 1 only
5. With reference to the Lassa fever, consider
(b) 2 only
the following statements:
(c) 1 and 3 only
1. Lassa fever is one of the hemorrhagic
(d) 1, 2 and 3
fever viruses.
2. Like the Ebola virus, Lassa fever is
2. With reference to recently launched web-
contagious from person to person.
based portal called „Digital Ocean‟, consider
3. Lassa virus is typically transmitted by
the following statements:
1. It is a web-based application for ocean the urine or feces of Mastomys rats to

data management. humans.


2. It has been developed under the aegis of Which of the statements given above is/are
Department of Science & Technology. correct?
Which of the statements given above is/are (a) 1 and 2 only
correct? (b) 2 and 3 only
(a) 1 only (c) 1 and 3 only
(b) 2 only (d) 1, 2 and 3
(c) Both 1 and 2
(d) Neither 1 nor 2 6. With reference to hypermetropia, which of
the following statements is/are correct?
3. With reference to 3D Bioprinting, consider 1. In Hypermetropia, parallel rays of light
the following statements: coming from infinity are focused behind
1. It is a form of additive manufacturing the retina.
that uses cells and other biocompatible 2. The people with hypermetropia have a
materials as “inks”. flatter cornea.
2. It prints living structures layer-by-layer 3. Hypermetropia is corrected using a
which mimic the behavior of natural concave lens.
living systems.
Select the correct answer using the code
Which of the statements given above is/are
given below.
correct?
(a) 2 only
(a) 1 only
(b) 1, 2 and 3
(b) 2 only
(c) 1 and 2 only
(c) Both 1 and 2
(d) 3 only
(d) Neither 1 nor 2
2 ©Vision IAS
7. Consider the following pairs: 10. Which of the following are the preconditions
Noble Gas Use for the occurrence of the annular Solar
1. Helium : Balloons Eclipse?

2. Krypton : Fog lights 1. The moon should be the farthest from

3. Xenon : Anesthetic the Earth.

Which of the pairs given above is/are 2. There should be a new moon condition.

correctly matched? 3. The Moon should be at or very near a

(a) 1 only lunar node.

(b) 2 only Which of the statements given above are


(c) 1 and 3 only correct?
(d) 1, 2 and 3 (a) 1 and 2 only
(b) 2 and 3 only
8. Which of the following phenomena are due (c) 1 and 3 only
to refraction of light? (d) 1, 2 and 3
1. A straw appearing to be bent when
submerged in a glass of liquid. 11. Consider the following statement with
2. A swimming pool appearing shallower refence to the Food and Agriculture
than it actually is. Organisation‟s (FAO) Food Price Index
3. The stars appear to be twinkling in the (FFPI):
night. 1. It is a measure of the annual change in
Select the correct answer using the code international prices of a basket of food
given below. commodities.
(a) 1 and 2 only 2. It uses prices of 2010-11 as base year for
(b) 2 and 3 only its calculations.
(c) 1 and 3 only Which of the statements given above is/are
(d) 1, 2 and 3 correct?
(a) 1 only
9. Consider the following statements: (b) 2 only
1. Chloroplasts are green-colored plastids, (c) Both 1 and 2
which comprise pigments called (d) Neither 1 nor 2
chlorophyll.
2. Chloroplasts are found in all green 12. Bowman‟s capsule and glomerulus are
plants and algae.
associated with which organ of the human
Which of the statements given above is/are
body?
correct?
(a) Liver
(a) 1 only
(b) Kidney
(b) 2 only
(c) Heart
(c) Both 1 and 2
(d) Lung
(d) Neither 1 nor 2
3 www.visionias.in ©Vision IAS
13. „Singularity‟, „Ergosphere‟ and 16. Which of the statements is/are correct
„Schwarzschild radius‟, astronomical terms regarding the recently conducted Coastal
Defence Exercise – Sea Vigil 21?
sometimes mentioned in the news are related
1. It is a tri–service exercise involving
to
assets of the Indian Navy, Indian Army
(a) Exoplanets and Indian Air Force.
(b) Asteroids 2. It is undertaken along the entire
(c) Mini satellites coastline including the Exclusive
Economic Zone (EEZ) of the country.
(d) Blackholes
Select the correct answer using the code
given below.
14. Consider the following statements: (a) 1 only
1. A cloned animal is not the same as a (b) 2 only
(c) Both 1 and 2
genetically modified animal.
(d) Neither 1 nor 2
2. A cloned animal shares the same DNA
as another animal, while a genetically 17. Consider the following statements with
modified animal has had a change made reference to the intermolecular interactions
to its DNA. between the different states of matter:
1. Intermolecular forces tend to keep the
Which of the statements given above is/are
molecules of different states of matter
correct?
together.
(a) 1 only 2. Thermal energy of the molecules tends
(b) 2 only to keep the molecules of different states
(c) Both 1 and 2 of matter apart.
Which of the statements given above is/are
(d) Neither 1 nor 2
correct?
(a) 1 only
15. Consider the following statements with (b) 2 only
reference to fuel cells: (c) Both 1 and 2
(d) Neither 1 nor 2
1. In fuel cells, hydrogen-rich fuel and
oxygen are used to generate electricity
18. Consider the following statements regarding
2. Fuel cells produce electricity in the form an adult stem cell:
of direct current (DC) 1. They can make identical copies of
3. Hydrogen fuel cells are more energy- themselves for long periods of time.
2. They are rare and their primary
efficient compared to diesel engines.
functions are to maintain the steady-state
Which of the statements given above is/are
functioning of a cell called homeostasis.
correct? Which of the statements given above is/are
(a) 1 only correct?
(b) 2 and 3 only (a) 1 only
(b) 2 only
(c) 1 and 2 only
(c) Both 1 and 2
(d) 1, 2 and 3 (d) Neither 1 nor 2
4 ©Vision IAS
19. With reference to coolant in a nuclear 22. The term 'Geosmin', recently seen in the
reactor, consider the following statements: news is:
1. Coolants majorly aim to remove and (a) A material recovered by Japan from a
transfer the amount of heat.
small near-Earth asteroid Itokawa.
2. India‟s First Prototype Fast Breeder
(b) An allotrope of carbon stronger than
Reactor will use gaseous Helium
Graphene
coolant.
Which of the statements given above is/ are (c) An organic compound produced by
correct? bacteria in the first rain after a dry
(a) 1 only period
(b) 2 only (d) An organic compound developed by
(c) Both 1 and 2 TERI to lower acidic levels of soil
(d) Neither 1 nor 2

23. Consider the following statements with


20. Recently, four sites in India have received
reference to Reverse Osmosis (RO):
the tag of World Heritage Irrigation
Structure (WHIS). In this context consider 1. In RO the total dissolved solids (TDS) in
the following statements: water, which covers trace chemicals,
1. UNESCO annually recognizes irrigation viruses, bacteria and salts can be reduced
structures of international significance to meet potable water standards.
and designate them as WHIS. 2. The UV radiation in RO quickens the
2. One of the criteria for recognition as a
sedimentation of solid particles, removes
WHIS is that structure must be 100 years
turbidity and improves the clarity of the
old.
water.
Which of the statements given above is/are
correct? Which of the statements given above is/are
(a) 1 only correct?
(b) 2 only (a) 1 only
(c) Both 1 and 2 (b) 2 only
(d) Neither 1 nor 2
(c) Both 1 and 2
(d) Neither 1 nor 2
21. With reference to "Aqua regia", consider the
following statements:
1. It is a mixture of concentrated 24. Recently, a team of archaeologists has

hydrochloric and nitric acid. discovered which seems to be the world‟s


2. It can dissolve gold. oldest cave painting in a remote valley on
Which of the statements given above is/are the island of Sulawesi, located at:
correct? (a) Philippines
(a) 1 only
(b) Indonesia
(b) 2 only
(c) Australia
(c) Both 1 and 2
(d) Brunei
(d) Neither 1 nor 2
5 www.visionias.in ©Vision IAS
25. The scientific name for Human beings is 28. Which among the below organisation
Homo sapiens Consider the following publishes annual Emissions Gap Report?
statements regarding Homosapiens:
(a) Intergovernmental Panel on Climate
1. Homo indicates the class to which
Change
human beings belong.
2. Sapiens indicates the species to which (b) World Meteorological Organization

human beings belong. (c) United Nations Environment Programme


3. All human beings belong to the (d) Food and Agriculture Organization
kingdom Animalia.
Which of the statements given above is/are
29. With reference to black holes, consider the
correct?
(a) 1 and 3 only following statements:

(b) 2 only 1. A black hole contains a huge amount of


(c) 2 and 3 only mass within a relatively small volume.
(d) 1, 2 and 3 2. Region of space beyond the black hole
called the event horizon
26. The celestial alignment of which two planets
3. Stars like our sun, upon its death, are
has been referred as „The Great Conjunction‟
which was seen recently? expected to become black holes.
(a) Jupiter and Saturn Which of the statements given above is/are
(b) Earth and Mars correct?
(c) Mercury and Venus
(a) 3 only
(d) Jupiter and Mars
(b) 1 and 2 only

27. Consider the following statements regarding (c) 1 and 3 only


evolution: (d) 1, 2 and 3
1. Lamarckism is the idea that an organism
can pass one characteristic that it
30. It is a cosmic phenomenon which is result of
acquired during its lifetime to its
magnification of light due to presence of
offspring.
2. Loss of limbs in snakes is explained by massive galaxy or cluster of galaxies in the

the theory of use and disuse of organs. universe. It is expected to help scientists in
3. Lamarck's theory of Inheritance of understanding the star formation.
Acquired Characteristics has been Which of the following best describes the
supported by Darwin's theory.
above given phenomena?
Which of the statements given above is/are
(a) Geotail
correct?
(a) 1 only (b) Spaghettification
(b) 2 and 3 only (c) Supernova explosion
(c) 1, 2 and 3 (d) Gravitational Lensing
(d) 1 and 2 only
6 www.visionias.in ©Vision IAS
31. Recently, the Sports Ministry has approved 34. “Athlete‟s Foot” is a disease caused by
the inclusion of four Indigenous Games to be (a) Bacteria
a part of Khelo India Youth Games 2021. In
(b) Fungus
this context consider the following pairs:
Indigenous games States (c) Protozoan
1. Gatka : Gujarat (d) Nematode
2. Kalaripayattu : Kerala
3. Thang – Ta : Manipur
35. Where is DNA contained in the Human
Which of the pairs given above is/are
correctly matched? body?
(a) 1 and 2 only 1. Blood
(b) 2 and 3 only
2. Urine
(c) 1, 2 and 3
(d) 1 and 3 only 3. Brain cells

4. Saliva
32. Which of the following will turn a Light 5. Feces
Blue litmus paper into red?
Select the correct answer using the code
1. Vinegar
2. Human Blood given below.
3. Baking Soda (a) 1, 2 and 4 only
4. Orange Juice
(b) 2 and 3 only
Select the correct answer using the code
(c) 1, 3, 4 and 5 only
given below.
(a) 1 and 2 only (d) 1, 2, 3, 4 and 5
(b) 2 and 3 only
(c) 1 and 3 only
36. An ELISA test may be used to diagnose:
(d) 1 and 4 only
1. HIV

33. Consider the following statements regarding 2. Lyme Disease


plants tissue: 3. Rotavirus
1. Phloem moves water and minerals
4. Syphilis
obtained from the soil.
2. Xylem transports products of 5. Pernicious anemia
photosynthesis from the leaves to other Select the correct answer using the code
parts of the plant.
given below.
Which of the statements given above is/are
(a) 1, 2 and 3 only
correct?
(a) 1 only (b) 2, 3 and 4 only
(b) 2 only (c) 1, 4 and 5 only
(c) Both 1 and 2
(d) 1, 2, 3, 4 and 5
(d) Neither 1 nor 2
7 www.visionias.in ©Vision IAS
37. In the context of Natural Capital Accounting 40. Which of the following are used in nuclear
and Valuation of Ecosystem Services fusion?
(NCAVES), consider the following
1. Deuterium
statements:
1. It has been jointly launched by United 2. Tritium
Nations Statistics Division, the United 3. Zirconium
Nations Environment Programme, the Select the correct answer using the code
Secretariat of the Convention on
given below.
Biological Diversity, and the European
(a) 1 and 3 only
Union.
2. India is one of the five partner countries (b) 2 and 3 only
of NCAVES. (c) 1 and 2 only
3. It is being funded by Green Climate (d) 1, 2 and 3
Fund
Which of the statements given above are
correct? 41. Consider the following statements with
(a) 1 and 2 only reference to electric fuse:
(b) 2 and 3 only 1. The principle behind the operation of the
(c) 1 and 2 only
fuse is the heating effect of the electrical
(d) 1, 2 and 3
current.
38. Which of the following methods can be 2. It is connected in parallel with the
applied for the prevention of corrosion? electric component.
1. Painting
Which of the statements given above is/are
2. Galvanising
3. Chrome Plating correct?
4. Anodising (a) 1 only
Select the correct answer using the code (b) 2 only
given below.
(c) Both 1 and 2
(a) 1 and 2 only
(d) Neither 1 nor 2
(b) 2, 3 and 4 only
(c) 1, 2, 3 and 4
(d) 1, 2 and 3 only 42. Which of the following organisms have the
chemoheterotrophic nature of nutrition?
39. With reference to the “fast breeder reactor‟
1. Bread moulds
(FBR), consider the following statements:
1. It generates more nuclear fuels than it 2. Yeast
consumes. 3. Mushrooms
2. It does not use moderators. 4. Cyanobacteria
3. It is the second stage of India‟s three
Select the correct answer using the code
stage nuclear programme.
Which of the statements given above are given below.
correct? (a) 1 and 2 only
(a) 1 and 2 only (b) 2 and 4 only
(b) 1 and 3 only
(c) 1, 3 and 4 only
(c) 2 and 3 only
(d) 1, 2 and 3
(d) 1, 2 and 3
8 www.visionias.in ©Vision IAS
43. With reference to neutrino, consider the 46. With reference to the Trans fats, consider the
following statements: following statements:
1. Neutrino is a tiny elementary particle 1. Trans fats, or trans-fatty acids, are
with no charge.
saturated fatty acids.
2. Neutrinos are the most abundant
2. Partially hydrogenated oils (PHO) are
particles in the universe.
3. Neutrinos can be produced in the the main source of industrially-produced
laboratory even though they are trans fat.
naturally occurring. 3. High trans fat intake increases the risk of
Which of the statements given above is/are coronary heart disease.
correct?
Which of the statements given above are
(a) 1 only
correct?
(b) 1 and 3 only
(c) 2 and 3 only (a) 1 and 2 only
(d) 1, 2 and 3 (b) 2 and 3 only
(c) 1 and 3 only
44. Consider the following statements with (d) 1, 2 and 3
reference to Epigenetic changes:
1. These are heritable changes in the
47. Which of the following is used in a
genotype of an organism that does not
lead to any phenotypic changes. fluorescent lamp to produce visible light?

2. These changes are irreversible in nature. (a) Mercury vapour


Which of the statements given above is/are (b) Phosphor
correct? (c) Helium
(a) 1 only (d) Astatine
(b) 2 only
(c) Both 1 and 2
48. Consider the following statements with
(d) Neither 1 nor 2
reference to Geostationary orbit and
45. Which of the following are the uses of DNA satellites:
fingerprinting? 1. Geostationary satellites can only be
1. For criminal identification placed above the Equator and at an
2. To resolve disputes of maternity/
altitude of 35,790 km.
paternity
2. Geostationary orbit requires a parabolic
3. To identify mutilated remains
4. In cases of exchange of babies in antenna.
hospital ward 3. There is only one geostationary orbit.
5. In forensic wildlife Which of the statements given above is/are
Select the correct answer using the code correct?
given below.
(a) 1 and 2 only
(a) 1, 2 and 5 only
(b) 3 only
(b) 2, 3 and 4 only
(c) 1, 3, 4 and 5 only (c) 1 and 3 only

(d) 1, 2, 3, 4 and 5 (d) 1, 2 and 3

9 www.visionias.in ©Vision IAS


49. Which of the following are bacterial 52. Consider the following statements regarding
diseases? Great Green Wall (GGW) initiative:
1. Diphtheria 1. It is an Australia‟s flagship initiative to
2. Tetanus combat coral bleaching of Great Barrier
Reef.
3. Leprosy
2. It is being funded by Green Climate
4. Pertussis (whooping cough)
Fund.
Select the correct answer using the code
Which of the statements given above is/are
given below. correct?
(a) 1, 2 and 4 only (a) 1 only
(b) 2, 3 and 4 only (b) 2 only
(c) 1 and 3 only (c) Both 1 and 2
(d) 1, 2, 3 and 4 (d) Neither 1 nor 2

50. Which of the following is/are the advantages 53. Atomic species namely protium, deuterium
and tritium are isotopes of which of the
of Lithium-ion batteries over traditional
following elements?
batteries such as Nickel-cadmium (NiCad)
(a) Carbon
batteries? (b) Hydrogen
1. Lithium-ion batteries are smaller in size, (c) Helium
require less maintenance, and are (d) Nitrogen
environmentally safer.
2. Lithium-ion batteries are cheaper. 54. With reference to 'PUSA decomposer'
3. Lithium-ion has virtually no self- recently seen in the news, which of the
discharge. following is correct?
Select the correct answer using the code (a) It is a fungi-based liquid solution for
stubble burning.
given below.
(b) It is a method to increase efficiency of
(a) 1 and 2 only
the incineration.
(b) 1 and 3 only (c) It is an enhanced fertilizer for increasing
(c) 2 and 3 only rice productivity.
(d) 1, 2 and 3 (d) None of the above.

51. Bacteriophages have been found to have 55. Recently a two-day water bird census was
numerous applications in medical conducted in Andhra Pradesh as part of
biotechnology. What are they? Asian Waterbird census – 2021. In this
context, consider the following statement
(a) They are heterotrophic microorganisms
with regard to Asian Waterbird Census
which obtain nutrients from the
(AWC):
consumption of bacteria.
1. It is a biennial exercise aimed at
(b) They are synthetic species of bacterium monitoring the wetlands and waterbirds
which infects virus leading to the death in the designated areas.
of the virus. 2. Both natural and man-made wetlands are
(c) They are viruses which parasitize a covered during the census.
bacterium by infecting it and Which of the statements given above is/are
reproducing inside it. correct?
(d) They are bacteria which infect other (a) 1 only
(b) 2 only
bacteria and form a symbiotic
(c) Both 1 and 2
relationship to survive.
(d) Neither 1 nor 2
10 www.visionias.in ©Vision IAS
56. Consider the following statements with 59. „SolarWinds hack‟, a term is sometime seen
reference to free-space optical in news is/are:
communication (FSO): (a) solar winds emanating from the Sun.
1. Free-space optical communication is an
(b) NASA‟s solar probe mission.
optical communication technology that
(c) European Space Agency's biggest solar
uses light propagating in free space to
wirelessly transmit data for telescope.
telecommunication. (d) a cyber attack against USA government
2. FSO is a line-of-sight technology. agencies.
3. Infrared Data Association (IrDA)
technology is a very simple form of free- 60. Which of the following is correct regarding
space optical communications.
the objective of the recently launched
Which of the statements given above are
SAATHEE portal?
correct?
(a) 1 and 2 only (a) It is a tracking system for the missing
(b) 1, 2 and 3 and found children.
(c) 1 and 3 only (b) It is a portal to have information of
(d) 2 and 3 only NGOs working especially for providing
educating the rural youth in the villages.
57. With reference to the cryogenic engine,
(c) It is a portal to provide banking services
consider the following statements:
to unorganised sector by linking Self
1. Cryogenic engines use liquid oxygen
and liquid hydrogen as propellants. Help Groups and banks through online
2. They are usually prepared for the upper platform.
stages (last stage) of the rocket. (d) It is a Management Information System
Which of the statements given above is/are (MIS) portal to monitor the progress of
correct? implementation of various Energy
(a) 1 only
Conservation programs at State level.
(b) 2 only
(c) Both 1 and 2
61. Consider the following statements with
(d) Neither 1 nor 2
reference to halogens:
58. With reference to the Lab-grown meat 1. All are non-metallic elements
consider the following statements: 2. They are widely used as water-
1. It is a type of meat that is prepared from purification agents and as pesticides
the regeneration of cell animal cultures 3. Reactive halogens are primarily
in a controlled lab environment.
responsible for the ozone hole in the
2. Consumption of lab-grown meat over
polar regions.
traditional meat is more beneficial for
the environment. Which of the statements given above is/are
Which of the statements given above is/are correct?
correct? (a) 1 only
(a) 1 only (b) 2 only
(b) 2 only (c) 1 and 3 only
(c) Both 1 and 2
(d) 1, 2 and 3
(d) Neither 1 nor 2
11 ©Vision IAS
62. In the context of recently launched Digital 65. Consider the following statements with
Payment Index (DPI), consider the following reference to radioactive emissions:
statements: 1. All radioactive emissions are electrically
1. It will measure deepening and neutral.
penetration of digital payments in the 2. Among the radioactive emissions,
country over different time periods. gamma rays have the highest penetration
2. It will be published by Reserve bank of power
India (RBI) on quarterly basis. Which of the statements given above is/are
Which of the statements given above is/are correct?
correct? (a) 1 only
(a) 1 only (b) 2 only
(b) 2 only (c) Both 1 and 2
(c) Both 1 and 2 (d) Neither 1 nor 2
(d) Neither 1 nor 2
66. Consider the following statements:
63. Which of the following is correct with 1. Dissolved oxygen in water is low as
regard to Bose-Einstein condensate? compared to the amount of oxygen in the
(a) It is the fifth state of matter. air.
(b) It is a range of subatomic particles 2. The rate of breathing in aquatic
responsible for giving matter different organisms is slower than the terrestrial
properties. organisms.
(c) It is a group of elementary particles Which of the statements given above is/are
forming basic building block of the correct?
matter. (a) 1 only
(d) An electrically conducting medium (b) 2 only
produced when the atoms in a gas (c) Both 1 and 2
become ionized. (d) Neither 1 nor 2

64. Geospatial Technology is often in news, it 67. With reference to Germline gene therapy
has applications in which of the following (GGT), consider the following statements:
areas? 1. In GGT germ cells are modified by the
1. Land-use and land-cover analysis introduction of functional genes into
2. Early warning and response during the their genomes.
disaster 2. This type of therapy allows for the
3. Mapping soil-erosion intensity correction of disease-causing gene
4. Mapping and analysing spread patterns variants that passed down from
of infectious diseases generation to generation.
Select the correct answer using the code Which of the statements given above is/are
given below. correct?
(a) 1 and 3 only (a) 1 only
(b) 2, 3 and 4 only (b) 2 only
(c) 1, 2 and 4 only (c) Both 1 and 2
(d) 1, 2, 3 and 4 (d) Neither 1 nor 2

12 www.visionias.in ©Vision IAS


68. An exploration carried out by Geological 71. Capillary action is defined as the motion of
Survey of India (GSI) has found promising liquids inside very narrow spaces without the
concentration of Vanadium metal in
assistance of, and most of the time in
Arunachal Pradesh. In this context, consider
opposition to, external forces such as
the following statements regarding
Vanadium: gravity. In this context, which of the
1. It is a high value metal used in following phenomena involve capillary
strengthening steel and titanium. action?
2. India is the largest consumer of this 1. A sponge absorbing water.
metal.
2. Burning of a candle.
Which of the statements given above is/are
correct? 3. Roots of plants drawing water from the
(a) 1 only soil.
(b) 2 only Select the correct answer using the code
(c) Both 1 and 2 given below.
(d) Neither 1 nor 2
(a) 1 only
(b) 2 only
69. In the context of Indian economy, which of
the statements given below is correct (c) 1 and 2 only
regarding K-shaped economic recovery? (d) 1, 2 and 3
(a) It happens when different sections of an
economy recover at starkly different
72. „BBX11‟, a term is sometimes seen in news
rates.
in the context of
(b) It happens when the economic growth
recovers sharply and returns to the path (a) a gene that facilitates in the greening of
it was following before the disruption plants
(c) It happens when a steep economic (b) a rare metal asteroid
decline is followed by a long period with (c) an exoplanet nearest to earth
no growth.
(d) a black hole in the galaxy
(d) It happens when the economy moves
beyond a recession into a period of
recovery before falling back down again 73. With reference to Benzo(a)pyrene, consider
into another recession. the following statements:
1. It is released due to the incomplete
70. Recently, Dhokra crafts were added to
combustion of organic matter.
Tribes India e-marketplace. In this context,
2. It is found in grilled and smoked meat
which of the following statements given
below is correct regarding Dhokra art? and fish.
1. It is practised by the tribals of eastern 3. It is a monitored pollutant under the
India. National Ambient Air Quality Standards
2. It is a non-ferrous metal casting style
Which of the statements given above is/are
which uses the lost-wax technique.
correct?
Select the correct answer using the code
given below: (a) 1 and 2 only
(a) 1 only (b) 2 only
(b) 2 only (c) 1 and 3 only
(c) Both 1 and 2 (d) 1, 2 and 3
(d) Neither 1 nor 2
13 www.visionias.in ©Vision IAS
74. Consider the following statements with 77. Consider the following statements:
reference to Biomineralization: 1. It is a critical structural component of
1. It is a process of a complete conversion the chlorophyll molecule.
of organic substances to inorganic
2. It is used for fruit and nut formation and
derivatives by living organisms.
essential for the germination of seeds.
2. Production of bones, shells and teeth in
organisms is a process of 3. Its deficiency shows yellowing between
biomineralization. veins of older leaves and leaves may
Which of the statements given above is/are drop.
correct? Which of the following essential plant
(a) 1 only
macro-nutrients do the above statements
(b) 2 only
refers to?
(c) Both 1 and 2
(d) Neither 1 nor 2 (a) Magnesium
(b) Sulphur
75. With reference to gravitational waves, which (c) Calcium
of the following statements is/are correct? (d) Iron
1. All gravitational waves are as old as the
Big Bang itself.
78. Consider the following statements with
2. Gravitational waves travel at the speed
reference to thorium:
of sound and squeeze and stretch
anything in their path. 1. Thorium is far more abundant in nature
3. 3.The first gravitational wave was than uranium.
detected by LIGO only in 2015. 2. Taken in equal amounts, Thorium can
Select the correct answer using the code generate more energy compared to
given below.
natural uranium.
(a) 3 only
Which of the statements given above is/are
(b) 1 and 2 only
(c) 2 and 3 only correct?
(d) 1, 2 and 3 (a) 1 only
(b) 2 only
76. Which of the following are base quantities as (c) Both 1 and 2
per the SI (International System of Units)?
(d) Neither 1 nor 2
1. Mass
2. Length
3. Charge 79. The words, Jiuzhang, Tangle Lake and

4. Temperature Sycamore sometimes mentioned in the news,


Select the correct answer using the code are related to
given below. (a) Mini satellites
(a) 1, 2 and 3 only (b) cyber malwares
(b) 1 and 2 only
(c) Craters on the surface of moon
(c) 1, 2 and 4 only
(d) Quantum technology
(d) 3 and 4 only

14 www.visionias.in ©Vision IAS


80. High Burden to High Impact (HBHI)‟, an 84. With reference to Graphene, consider the
Initiative of World health organisation is following statements:
1. It is a three-dimensional carbon
started for?
allotrope.
(a) Judicious use of antibiotics
2. It is made up of very tightly bonded
(b) Fight against Malaria carbon atoms organized into a hexagonal
(c) Reducing the cases of Tuberculosis lattice.
(d) Expanding Programme on Immunization 3. It has very high electrical conductivity
Which of the statements given above is/are
correct?
81. Consider the following statements with
(a) 1 and 2 only
reference to the adiabatic process: (b) 2 and 3 only
1. It is a process in which there is no (c) 1 and 3 only
transfer of heat between the system and (d) 1, 2 and 3
surroundings.
85. Which of the following tests are being used
2. These processes lead to a decrease in
for testing COVID-19?
entropy.
1. Rapid Antigen Test (RAT)
Which of the statements given above is/are 2. Reverse transcription polymerase chain
correct? reaction (RT-PCR) test
(a) 1 only 3. CBNAAT test
4. Serology Test
(b) 2 only
5. TrueNat test
(c) Both 1 and 2
Select the correct answer using the code
(d) Neither 1 nor 2 given below.
(a) 1, 2, and 4 only
82. With reference to chemistry, the (b) 1, 2, and 3 only
(c) 4 and 5 only
intermolecular forces interacting between
(d) 1, 2, 3, 4 and 5
particles (atoms and molecules) is referred to
as: 86. Consider the following statements with
(a) Electrostatic force reference to electromagnetic radiation:
(b) Weak nuclear force 1. Waves with shorter wavelengths have
(c) Van der Waals force more energy.
2. X-rays have a higher frequency than
(d) Gravitational force
gamma rays.
3. All electromagnetic radiations travel at
83. In chemistry, the Octet rule is related to the same speed through the vacuum.
(a) Reactivity series of metals. Which of the statements given above is/are
(b) Arrangement of elements in the periodic correct?
(a) 3 only
table.
(b) 2 and 3 only
(c) Arrangement of electrons in the element.
(c) 1 and 3 only
(d) Energy release during nuclear fission. (d) 1, 2 and 3

15 www.visionias.in ©Vision IAS


87. Consider the following statements: 90. Consider the following statements with
1. Pneumatophores are the roots that reference to transverse waves:
facilitate a gas exchange under anaerobic 1. In transverse waves, the particles are
soil conditions. displaced parallel to the direction the
2. Haustorial roots are the roots of parasitic wave travels.
plants that can absorb water and 2. Mechanical transverse waves cannot
nutrients from another plant. propagate in gaseous media.
Which of the given statements given above
3. Secondary seismic waves are a type of
is/are correct?
transverse waves.
(a) 1 only
Which of the statements given above is/ are
(b) 2 only
correct?
(c) Both 1 and 2
(a) 1 only
(d) Neither 1 nor 2
(b) 1 and 2 only
(c) 2 and 3 only
88. Which of the following statements with
(d) 1, 2 and 3
reference to allotropes is/are correct?
1. It is the property of some chemical
elements to exist in two or more 91. Digestion of lipids takes place in the
different forms, in the different physical presence of which of the following?
state 1. Pepsin
2. Diamond and fullerenes are allotropes of 2. Bile
carbon 3. Lipases
Select the correct answer using the code Select the correct answer using the code
given below. given below.
(a) 1 only (a) 1 and 2 only
(b) 2 only (b) 2 and 3 only
(c) Both 1 and 2 (c) 1 and 3 only
(d) Neither 1 nor 2 (d) 1, 2 and 3

89. Consider the following statements: 92. Which of the following is/are an example of
1. Gypsum on hydration reaction forms
zoonotic disease?
Plaster of Paris
1. Anthrax
2. Gypsum application improves soil
2. Plague
aeration and water percolation through
3. Brucellosis
the soil profile
4. Salmonellosis
3. It reduces the runoff of phosphorous
5. West Nile fever
from agricultural fields.
Select the correct answer using the code
Which of the statements given above is/are
given below.
correct?
(a) 1 only (a) 1, 2, 3 and 4 only

(b) 1 and 2 only (b) 2, 3, 4 and 5 only


(c) 2 and 3 only (c) 1 and 5 only
(d) 1, 2 and 3 (d) 1, 2, 3, 4 and 5

16 www.visionias.in ©Vision IAS


93. Consider the following statements with 96. In the context of the Climate Change
reference to the International Union of Pure Performance Index (CCPI), consider the
and Applied Chemistry (IUPAC): following statements:
1. It is the sole organization with the 1. It is released annually by the United
responsibility for updating the Periodic Nations Environment Program (UNEP)
Table.
2. It assesses countries‟ performance in
2. It does not have the mandate to name
four categories - GHG emissions,
new elements and compounds.
renewable energy, energy use and
Which of the statements given above is/are
climate policy.
correct?
3. India is among the top 10 countries in
(a) 1 only
(b) 2 only the CCPI 2020.
(c) Both 1 and 2 Which of the statements given above is/are
(d) Neither 1 nor 2 correct?
(a) 2 only
94. Consider the following statements with (b) 2 and 3 only
reference to radio waves: (c) 1 and 3 only
1. It has the longest wavelengths out of all (d) 1, 2 and 3
known waves in the electromagnetic
spectrum. 97. Which of the following chemical reactions
2. Wi-fi uses radio waves to transmit are exothermic in nature?
information between the device and a
1. Respiration
router.
2. Decomposition of organic matter
3. Radio waves cannot diffract and tend to
3. Photosynthesis
go straight.
Select the correct answer using the code
Which of the statements given above is/are
given below.
not correct?
(a) 1 only (a) 1 only
(b) 1 and 2 only (b) 2 only
(c) 3 only (c) 1 and 3 only
(d) 1, 2 and 3 (d) 1, 2 and 3

95. Consider the following statements with 98. Which among the following is/ are present
reference to the 'Artificial Sun', recently seen only in plant cells?
in the news: 1. Plastids
1. It is a superconducting fission device. 2. Vacuoles
2. China is the only county that has
3. Mitochondria
developed and used this technology.
Select the correct answer using the code
Which of the statements given above is/are
given below.
correct?
(a) 1 and 2 only
(a) 1 only
(b) 1 only
(b) 2 only
(c) Both 1 and 2 (c) 2 and 3 only

(d) Neither 1 nor 2 (d) 1, 2 and 3

17 www.visionias.in ©Vision IAS


99. The photoelectric effect is the emission of
electrons when electromagnetic radiation,
such as light, hits a material. The
photoelectric effect is used in which of the
following?
1. Solar Cells
2. Fibre Optic Cables
3. Pollution Monitoring
Select the correct answer using the code
given below.
(a) 1 and 2 only
(b) 3 only
(c) 2 and 3 only
(d) 1, 2 and 3

100. Consider the following statements regarding


Better than Cash Alliance (BTCA):
1. It is a partnership of governments,
companies, and international
organizations that accelerates the
transition from cash to responsible
digital payments
2. India is not a member of BTCA
Which of the statements given above is/are
correct?
(a) 1 only
(b) 2 only
(c) Both 1 and 2
(d) Neither 1 nor 2

Copyright © by Vision IAS


All rights are reserved. No part of this document may be reproduced, stored in a retrieval system or transmitted
in any form or by any means, electronic, mechanical, photocopying, recording or otherwise, without prior
permission of Vision IAS.
18 www.visionias.in ©Vision IAS
VISIONIAS
www.visionias.in
ANSWERS & EXPLANATIONS
GENERAL STUDIES (P) TEST – 3200 (2021)

Q 1.B
 Genetically modified crops (GM crops) are plants used in agriculture, the DNA of which has been
modified using genetic engineering techniques. In the late 1990s, German scientists developed a
genetically modified and biofortified crop variety of rice called Golden Rice.
 The Golden rice was created by transforming rice with two beta-carotene biosynthesis genes:
o psy (phytoene synthase) from daffodil ('Narcissus pseudonarcissus'), which is a variety of maize.
Hence statement 1 is not correct.
o crtI (phytoene desaturase) from the soil bacterium Erwinia uredovora.
 Thus, genetic modification of the rice resulted into in the greatest accumulation of total carotenoids and
beta‐carotene which is collectively termed provitamin A. Therefore, golden rice is rich source of Vitamin
A. It will help to fight against Vitamin A deficiency, which is the leading cause of blindness among
children and can also lead to death due to infectious diseases such as measles. Hence statement 2 is
correct.
 Indian government has not allowed any GM crops for the consumption. For the time being, Bt cotton is
the only genetically modified crop that is under cultivation in India. It is Philippines government that
approved Golden Rice for direct use 2019. Currently, Bangladesh government is thinking about approving
the cultivation of golden rice. Hence statement 3 is not correct.

Q 2.A
 Recent Context - The Ministry of Earth Sciences has inaugurated the web-based application ―Digital
Ocean‖ developed by INCOIS.
 Digital Ocean: Digital Ocean is a first of its kind digital platform for Ocean Data Management. Hence
statement 1 is correct.
o It has been developed by Indian National Centre for Ocean Information Services (INCOIS) under
Ministry of Earth Sciences. hence statement 2 is not correct.
o It includes a set of applications developed to organize and present heterogeneous oceanographic data
by adopting rapid advancements in geospatial technology.
o It facilitates an online interactive web-based environment for data integration, 3D and 4D (3D in
space with time animation) data visualization, data analysis to assess the evolution of oceanographic
features etc.
 Significance:
o It shares knowledge about the ocean with a wide range of users including research institutions,
operational agencies, strategic users, academic community, maritime industry and policy makers. It
will serve as a one stop-solution for all the data related needs of ocean of a wide range of users.
o It provides free access to information to the general public and the common man.
o It will play a central role in sustainable management of our oceans and expanding our ‗Blue
Economy‘ initiatives. It will help improve our understanding of working of oceans.
o It will be promoted as a platform for capacity building on Ocean Data Management for all Indian
Ocean Rim countries.
 Indian National Centre for Ocean Information Services (INCOIS):
o It is an autonomous body under the Ministry of Earth Sciences. It was established in 1999.
o It is a unit of the Earth System Science Organization (ESSO).
o It is mandated to provide the best possible ocean information and advisory services to society,
industry, government agencies and the scientific community through sustained ocean observations
and constant improvements through systematic and focused research.

1 www.visionias.in ©Vision IAS


Q 3.C
 What is 3D Bioprinting?
o 3D Bioprinting is a form of additive manufacturing that uses cells and other biocompatible
materials as ―inks‖, also known as bioinks, to print living structures layer-by-layer which mimic
the behavior of natural living systems. Hence both statements are correct.
o Bioprinted structures, such as an organ-on-a-chip, can be used to study functions of a human body
outside the body (in vitro), in 3D. The geometry of a 3D bioprinted structure is more similar to that of
a naturally occurring biological system than an in vitro study performed in 2D and can be more
biologically relevant. It‘s used most commonly in the fields of tissue engineering and bioengineering,
and materials science. 3D bioprinting is also increasingly used for pharmaceutical development and
drug validation, and in the future will be used for medical applications in clinical settings – 3D printed
skin grafts, bone grafts, implants, biomedical devices, and even full 3d printed organs are all active
topics of bioprinting research.
 How does 3D bioprinting work?
o 3D bioprinting starts with a model of a structure, which is recreated layer-by-layer out of a bio-ink
either mixed with living cells or seeded with cells after the print is complete. These starting models
can come from anywhere – a CT or MRI scan, a computer-generated design (CAD) program, or a file
downloaded from the internet.
o That 3D model file is then fed into a slicer – a specialized kind of computer program which analyzes
the geometry of the model and generates a series of thin layers, or slices, which form the shape of the
original model when stacked vertically. Cura and slic3r are examples of slicers commonly used in 3D
printing. Allevi also has a specialized slicer, optimized specifically for bioprinting, built into our
Allevi Bioprint software.
o Once a model is sliced, the slices are transformed into path data, stored as a g-code file, which can be
sent to a 3D bioprinter for printing. The bioprinter follows instructions in the g-code file in order,
including instructions to control for temperature of the extruders, extrusion pressure, bed plate
temperature, crosslinking intensity and frequency, and, of course, the 3D movement path generated by
the slicer. Once all of the g-code commands are completed, the print is done and can be cultured or
seeded with cells as part of a biostudy.
Q 4.B
 Nanomicelles are ultramicroscopic (size 10 – 100 nm) structures used in the treatment of
disease. They have a hydrophilic outer shell and a hydrophobic exterior, making them capable of
delivering poorly water-soluble drugs and protecting drug molecules.
 Intensive research has shown that nano micelles could be used in a variety of medico-pharmacological
instances to catalyze drug effects or improve treatment. Studies postulate that nanomicelles are helpful
in facilitating therapeutic solutions that involve protein and peptide delivery.
 Bioengineers have taken great interest in the functionality of nanomicelles due to their small size (10 –
100 nm), the capability to solubilize lipophilic drugs in various levels of drug loading, low toxicity,
and ability to conjugate with target ligand and stimuli-sensitive regions.
 However, because micelles disintegrate rapidly after tissue penetration, using nanomicelles as drug
carriers become a challenge. Research suggests that in order to efficiently address this, prolonged blood
circulation and control for the appropriate release of drugs on target sites should be modulated.
 Nanomicelles are also hypothesized to improve cancer treatment through overcoming drug resistance,
improving anti-cancer drug efficacy, and reducing drug toxicity. Hence the correct option is (b)
 Tardigrades - often called water bears - are creatures under a millimeter long that can survive being
heated to 150 Degree C and frozen to almost absolute zero. They were traveling on an Israeli spacecraft
that crash-landed on the moon in April 2019.
 Recently, China live-streamed footage of its new manned submersible parked at the bottom of the
Mariana Trench on Friday, part of a historic mission into the deepest underwater valley on the planet. The
"Fendouzhe", or "Striver", descended more than 10,000 meters (about 33,000 feet) into the submarine
trench in the western Pacific Ocean with three researchers on board.
 Also in a separate research, Eurythenes plasticus, a newly described species of crustacean from the
Marian Trench in the Pacific Ocean, was named after pieces of plastic ingested by the animal.

Q 5.C
 Lassa fever:
o Lassa fever is one of the hemorrhagic fever viruses like Ebola virus, Marburg virus, and
others. Hence, statement 1 is correct.

2 www.visionias.in ©Vision IAS


o It is caused by the Lassa virus, a member of the arenavirus family of viruses.
o Unlike the Ebola virus, Lassa fever is not as contagious from person to person, nor as deadly. Hence,
statement 2 is not correct.
o Lassa virus is typically transmitted by the urine or feces of Mastomys rats to humans. Hence,
statement 3 is correct.
o Health workers may be infected by direct contact with blood, body fluids, urine, or stool of a patient
with Lassa fever.
o Lassa fever occurs primarily in West Africa in areas where these rodents live.
 Blueprint Priority Diseases of WHO: Worldwide, the number of potential pathogens is very large, while
the resources for disease research and development (R&D) is limited. To ensure efforts under WHO‘s
R&D Blueprint are focused and productive, a list of diseases and pathogens are prioritized for R&D in
public health emergency contexts. A WHO tool distinguishes which diseases pose the greatest public
health risk due to their epidemic potential and/or whether there is no or insufficient counter-measures. At
present, the priority diseases are
o COVID-19
o Crimean-Congo hemorrhagic fever
o Ebola virus disease and Marburg virus disease
o Lassa fever
o Middle East respiratory syndrome coronavirus (MERS-CoV) and Severe Acute Respiratory
Syndrome (SARS)
o Nipah and henipaviral diseases
o Rift Valley fever
o Zika―
o Disease X‖: Disease X represents the knowledge that a serious international epidemic could be caused
by a pathogen currently unknown to cause human disease. The R&D Blueprint explicitly seeks to
enable early cross-cutting R&D preparedness that is also relevant for an unknown ―Disease X‖.

Q 6.C
 Hypermetropia is also known as far-sightedness. It is a form of refractive error in which parallel rays
of light coming from infinity are focused behind the light-sensitive layer of the retina when the eye is
at rest. Hence statement 1 is correct.
 A person with hypermetropia can see distant objects clearly but cannot see nearby objects distinctly. The
near point for the person is farther away from the normal near point. This is because in hypermetropia,
the cornea is flatter or the axial length is too short. Therefore, the images do not focus by the time they
reach the retina. Hence statement 2 is correct.
 This defect arises either because:
 the focal length of the eye lens is too long or
 the eyeball has become too small.
 This defect can be corrected by using a convex (outward facing) lens of appropriate power. Eyeglasses
with converging lenses provide the additional focusing power required for forming the image on the
retina. Hence statement 3 is not correct.

Q 7.D
 The p-Block Elements comprise those belonging to Group 13 to 18 and these together with the s-Block
Elements are called the Representative Elements or Main Group Elements.
 At the end of each period in a periodic table is a noble gas element with a closed valence shell. All the
orbitals in the valence shell of the noble gases are completely filled by electrons and it is very difficult
to alter this stable arrangement by the addition or removal of electrons. The noble gases thus exhibit
very low chemical reactivity.
 Noble gases have uses that are derived from their other chemical properties. The very low boiling points
and melting points of the noble gases make them useful in the study of matter at extremely low
temperatures. Therefore often used for specific industrial processes.
o The low solubility of helium in fluids leads to its admixture with oxygen for breathing by deep-
sea divers: because helium does not dissolve in the blood, it does not form bubbles upon
decompression (as nitrogen does, leading to the condition known as decompression sickness, or the
bends).
o Xenon has been used as an anesthetic; although it is costly, it is non-flammable and readily
eliminated from the body.

3 www.visionias.in ©Vision IAS


o Radon is highly radioactive; its only uses have been those that exploit this property (e.g.,
radiation therapy). (Oganesson is also radioactive, but, since only a few atoms of this element have
thus far been observed, its physical and chemical properties cannot be documented.).
o Krypton is used in fluorescent bulbs, flashbulbs, and lasers. Lamps filled with krypton are used at
some airports as approach lights since their light can penetrate dense fog unusually well.
o Neon is also used as fog lights. Hence option (d) is the correct answer.

Q 8.D
 Refraction is the bending of a wave when it enters a medium where its speed is different. The refraction
of light when it passes from a fast medium to a slow medium bends the light ray toward the normal to the
boundary between the two media. The amount of bending depends on the indices of refraction of the two
media.
 Some of the common examples are:
o A straw in a glass with water looks bent or broken.
o A swimming pool appears shallower than it actually is. This is because the light from the pool is
refracted away from the normal when moving from water to the air.
o The setting sun looks oval in shape because the light from the sun is refracted at a different rate
when passes through the atmosphere.
o The light of stars is refracted when passes through the different region in the atmosphere. The
angle of refraction varies a little from time to time. As a result, the stars look twinkling.

Q 9.C
 Chloroplasts: They are green-colored plastids, which comprise green-colored pigments within the plant
cell and are called chlorophyll. Hence, statement 1 is correct.
 Chloroplast is an organelle that contains the photosynthetic pigment chlorophyll that captures sunlight and
converts it into useful energy, thereby, releasing oxygen from water.
 Chloroplasts are found in all green plants and algae. They are the food producers of plants. These are
found in the guard cells located in the leaves of the plants. They contain a high concentration of
chlorophyll that traps sunlight. This cell organelle is not present in animal cells. Hence, statement 2 is
correct.
 Chloroplast has its own DNA and can reproduce independently, from the rest of the cell. They also
produce amino acids and lipids required for the production of chloroplast membrane.
 Functions of Chloroplast: Following are the important chloroplast function:
o The most important function of the chloroplast is to synthesize food by the process of photosynthesis.
o Absorbs light energy and converts it into chemical energy.
o Chloroplast has a structure called chlorophyll which functions by trapping the solar energy and used
for the synthesis of food in all green plants.
o Produces NADPH and molecular oxygen (O2) by photolysis of water.
o Produces ATP – Adenosine triphosphate by the process of photosynthesis.
o The carbon dioxide (CO2) obtained from the air is used to generate carbon and sugar during the
Calvin Cycle or dark reaction of photosynthesis.
o Where does the photosynthesis process occur in the plant cell?
o In all green plants, photosynthesis takes place within the thylakoid membrane of the Chloroplast.

4 www.visionias.in
Q 10.D
 Annular solar eclipse was observed on June 20-21, 2020 in some parts of India. An eclipse occurs when
the moon while orbiting the Earth, comes in between the sun and the Earth. Due to which the moon blocks
the sun‘s light from reaching the Earth, causing an eclipse of the sun or a solar eclipse.
 There are mainly three types of eclipses:
o Annular solar eclipse: In this type of eclipse, the moon does not block the sun completely, but looks
like a ―dark disk on top of a larger sun-colored disk‖ forming a ―ring of fire‖. The following are
preconditions for occurrence of Annular solar eclipse:
 There should be New Moon condition, moon is aligned between the Sun and Earth, blocking
the light received by the Earth from the Sun. Hence statement 2 is correct.
 The moon should be at or very near a lunar node so that the Sun, Moon and the Earth all
are in a straight line. Hence statement 3 is correct.
 The moon should be near the apogee (the farthest point of the Moon from Earth). Hence
statement 1 is correct.
o Total solar eclipse: It is visible only from a small area on Earth. According to NASA, people who are
able to view the total solar eclipse are in the centre of the moon‘s shadow as and when it hits the
Earth. A total solar eclipse happens when the sun, moon and Earth are in a direct line. • Following are
the preconditions for the occurrence of Total solar eclipse:
 Moon should be the closest point of the Moon from Earth (near perigee).
 The Moon should be (or very near) a lunar node, so the Earth, the Moon, and the Sun are aligned
in a straight (or nearly straight) line.
 There should be New Moon condition.
o Partial solar eclipse: During this eclipse, the shadow of the moon appears on a small part of the sun

Q 11.D
 Recent context – As per monthly FAO Food Price Index (FFPI), World food prices rose for a seventh
consecutive month in December 2020. For the whole of 2020, FFPI averaged 97.9 points, a three-year
high and a 3.1% increase from 2019.
 About FAO Food Price Index (FFPI)
o The FAO Food Price Index (FFPI) is a measure of the monthly change in international prices of a
basket of food commodities. Hence statement 1 is not correct.
o It consists of the average of five commodity group price indices weighted by the average export
shares of each of the groups over 2014-16. The current FFPI comprises separate sub-indices for five
major food commodity groups – i) Cereal Price Index, ii) Meat Price Index, iii) Sugar Price Index, iv)
Vegetable oil price index and v) Dairy Price Index. Hence statement 2 is correct.
o The base year of FPPI is 2014-16. Hence statement 2 is not correct.
 Food and Agricultural Organisation (FAO):
o It is a specialized agency of the United Nations that leads international efforts to defeat hunger. It was
founded in 1945. Its headquarter is located in Rome, Italy
o Its goal is to achieve food security for all and make sure that people have regular access to enough
high-quality food to lead active, healthy lives.

Q 12.B
 A nephron is the basic structural and functional unit of the kidney. They are the microscopic structure
composed of a renal corpuscle and a renal tubule.
5 www.visionias.in ©Vision IAS
 Structure of Nephron
o The mammalian nephron is a long tube-like structure, its length varying from 35–55 mm long. At one
end, the tube is closed, folded and expanded, into a double-walled, cuplike structure called the
Bowman‘s capsule or renal corpuscular capsule, which encloses a cluster of microscopic blood
vessels called the glomerulus. This capsule and glomerulus together constitute the renal corpuscle.
o Functions of Nephron
o The primary function of the nephron is removing all waste products including the solid wastes, and
other excess water from the blood, converting blood into urine, reabsorption, secretion, and excretion
of numerous substances.
o As the blood passes through the glomerulus with high pressure, the small molecules are moved into
the glomerular capsules and travel through a winding series of tubules.
o The cell present in each tube absorbs different molecules excluding the glucose, water, and other
beneficial molecules which are called ass ultrafiltrate. As the ultrafiltrate molecules travel down the
tubules they become more and more hypertonic, which results in more amount of water to be
extracted from the ultrafiltrate before it exits the nephrons.
o The blood surrounding the nephron travels back into the body through the renal blood vessels, which
are free of toxins and other excess substances. The obtained ultrafiltrate is urine, which travels down
via the collecting duct to the bladder, where it will be stored and released through the urethra.

Q 13.D
 A black hole is a place in space where gravity pulls so much that even light can not get out. The gravity is
so strong because matter has been squeezed into a tiny space. Black holes were predicted by the Einstein's
theory of general relativity, which showed that when a massive star dies, it leaves behind a small, dense
remnant core. If the core's mass is more than about three times the mass of the Sun, the force of gravity
overwhelms all other forces and produces a black hole.
 Basic parts of black hole:
o Singularity: It is the one-dimensional point in the centre of a black hole which contains a huge mass
in an infinitely small space, where density and gravity become infinite and space-time curves
infinitely. It is a tiny volume with very big density.
o The event horizon: It is the "point of no return" around the black hole. It is not a physical surface, but
a sphere surrounding the black hole that marks where the escape velocity is equal to the speed of light.
Its radius is the Schwarzschild radius. When is the matter inside the event horizon, It will fall to the
centre.
o The Schwarzschild Radius: This is the event horizon's radius. It is the radius at which the escape
velocity is equal to the speed of light, R = 2GM/c2 Scientists are actively engaged in research to better
understand what happens at these singularities, as well as how to develop a full theory that better
describes what happens at the center of a black hole.
o The Ergosphere: If the black hole is rotating, then as it spins, its mass causes the space time around
the black hole to rotate as well. This region is called the ergosphere.
o The Accretion Disk: This is a disk composed of stellar material that is spiraling towards that black
hole
 Hence option (d) is the correct answer.

Q 14.C
 What is a clone?
o A clone is a living organism (such as a plant or animal), which shares the same genetic information as
another organism. However, their characteristics can be affected by random mutations which occur in
their DNA during development in the womb or by the environment that they grow up in, so, although
clones have the same DNA, they may not look the same or behave in the same way.
o While some clones can be found in nature, it is also possible for scientists to create a clone or
identical copy of an organism. It is important to understand that a cloned animal is not the same as a
genetically modified animal. Hence, statement 1 is correct.
o A cloned animal shares the same DNA as another animal, while a genetically modified animal has had
a change made to its DNA, but does not share its DNA with any other animals. Hence, statement 2 is
correct.
o Scientists can also use bacteria or viruses to replicate or clone individual DNA sequences that they are
interested in. This is known as molecular or DNA cloning.

6 www.visionias.in ©Vision IAS


Q 15.D
 Fuel cells are used primarily for backup power for commercial, industrial and residential buildings.
A fuel cell uses the chemical energy of hydrogen or another fuel to cleanly efficiently produce
electricity. Hence statement 1 is correct.
 If hydrogen is the fuel electricity, water and heat are the only products.
 Fuel cells produce electricity in the form of direct current (DC), not in the form of Alternating current
(AC). Hence statement 2 is correct.
 Fuel cells are unique in terms of their potential applications, they can provide power for systems as large
as a utility power station and as small as a laptop computer.
 Hydrogen has a high energy density as compared to other fuels, thus it produces more energy in lesser
weight due to which it can prove to be a viable option for heavy vehicles covering long routes in the
future. Hence statement 3 is correct.

Q 16.B
 Recent Context - the second edition of the biennial pan-India coastal defence exercise ‗Sea Vigil-21‘ was
conducted on 12 – 13 January 2021. The inaugural edition of the exercise was conducted in January 2019.
It has been billed as India's largest coastal defence drill.
 Sea Vigil – 21: It will provide a realistic assessment of our strengths and weaknesses in the domain of
maritime security and coastal defence and thus will help in further strengthening maritime security and
coastal defence.
 It was undertaken along the entire 7516 km coastline and Exclusive Economic Zone of India. It involved
all the 13 coastal States and Union Territories along with other maritime stakeholders, including the
fishing and coastal communities. Hence statement 2 is correct.
 The exercise was coordinated by the Indian Navy. Assets of the Indian Navy, Coast Guard, Customs
and other maritime agencies participated in the exercise. The conduct of the exercise is also facilitated by
the Ministries of Defence, Home Affairs, Shipping, Petroleum and Natural Gas, Fisheries, Customs, State
Governments and other agencies of Centre/ State. Thus, it is not a tri-service exercise. Hence statement 1
is not correct.

Q 17.C
 Intermolecular forces tend to keep the molecules together but the thermal energy of the molecules
tends to keep them apart. Three states of matter are the result of a balance between intermolecular
forces and the thermal energy of the molecules. Hence the correct option is (c)
 When molecular interactions are very weak, molecules do not cling together to make liquid or solid unless
thermal energy is reduced by lowering the temperature.
 Gases do not liquify on compression-only, although molecules come very close to each other and
intermolecular forces operate to the maximum. However, when the thermal energy of molecules is
reduced by lowering the temperature; the gases can be very easily liquified.

Q 18.C
 What Is an Adult Stem Cell?
o Adult stem cells, like all stem cells, share at least two characteristics.
o First, they can make identical copies of themselves for long periods of time; this ability to proliferate
is referred to as long-term self-renewal. Hence, statement 1 is correct.
o Second, they can give rise to mature cell types that have characteristic morphologies (shapes) and
specialized functions.
o Typically, stem cells generate an intermediate cell type or types before they achieve their fully
differentiated state. The intermediate cell is called a precursor or progenitor cell. Progenitor or
precursor cells in fetal or adult tissues are partly differentiated cells that divide and give rise to
differentiated cells. Such cells are usually regarded as "committed" to differentiating along a
particular cellular development pathway, although this characteristic may not be as definitive as once
thought.
o Adult stem cells are rare. Their primary functions are to maintain the steady-state functioning of
a cell—called homeostasis—and, with limitations, to replace cells that die because of injury or
disease. Hence, statement 2 is correct.
o Furthermore, adult stem cells are dispersed in tissues throughout the mature animal and behave very
differently, depending on their local environment. In contrast, stem cells in the small intestine are
stationary and are physically separated from the mature cell types they generate.

7 www.visionias.in ©Vision IAS


o Unlike embryonic stem cells, which are defined by their origin (the inner cell mass of the
blastocyst), adult stem cells share no such definitive means of characterization. In fact, no one knows
the origin of adult stem cells in any mature tissue. Some have proposed that stem cells are somehow
set aside during fetal development and restrained from differentiating.
o Most of the information about adult stem cells comes from studies of mice. The list of adult tissues
reported containing stem cells is growing and includes bone marrow, peripheral blood, brain, spinal
cord, dental pulp, blood vessels, skeletal muscle, epithelia of the skin and digestive system, cornea,
retina, liver, and pancreas.
o In order to be classified as an adult stem cell, the cell should be capable of self-renewal for the
lifetime of the organism.
o Ideally, adult stem cells should also be clonogenic. In other words, a single adult stem cell should be
able to generate a line of genetically identical cells, which then gives rise to all the appropriate,
differentiated cell types of the tissue in which it resides.

Q 19.A
 A nuclear reactor coolant is a coolant in a nuclear reactor used to remove heat from the nuclear
reactor core and transfer it to electrical generators and the environment. Frequently, a chain of two
coolant loops is used because the primary coolant loop takes on short-term radioactivity from the
reactor. Hence statement 1 is correct.
 Molten salts share with metals the advantage of low vapour pressure even at high temperatures, and are
less chemically reactive than sodium. Salts containing light elements like FLiBe {a molten salt made from
a mixture of lithium fluoride (LiF) and beryllium fluoride (BeF2)} can also provide moderation. In the
Molten Salt Reactor Experiment, it served as a solvent carrying nuclear fuel.
 Gases have also been used as a coolant. Helium is extremely inert both chemically and with respect to
nuclear reactions but has a low heat capacity. But India‘s first Prototype Fast Breeder Reactor which
will be commissioned in December 2021 at Kalpakkam marking the second of the three-stage nuclear
power programme will use liquid sodium as a coolant. Hence statement 2 is not correct.
Q 20.B
 Recent context – Four sites in India have received the World Heritage Irrigation Structure (WHIS) tag
this year.
 World Heritage Irrigation Structure (WHIS): The International Commission on Irrigation and
Drainage (ICID) annually recognises irrigation structures of international significance. It is on the lines of
World Heritage Sites recognised by the UNESCO. Hence statement 1 is not correct.
 The main objectives of recognition as a ―World Heritage Irrigation Structures‖ are:
o Tracing the history of and understanding the evolution of irrigation in the civilizations across the
world.
o To select and collect information on historical irrigation structures from around the world, understand
their significant achievements and gather knowledge about the unique features that have sustained the
project for such a long period;
o To learn the philosophy and wisdom on sustainable irrigation from these structures; and
o To protect/preserve these historical irrigation structures
 Criterion for recognition -The type of the structures or facilities to be considered for recognition as
World Heritage Irrigation Structures shall fulfil following criterion:
o The structure shall be more than 100 years old; Hence statement 2 is correct.
o The structures shall fall under one of the following categories:(i) Dams (operational largely for
irrigation purpose), (ii) Water storage structures such as tanks for irrigation, (iii) Barrages and other
water diversion structures, (iv) Canal Systems, (v) Old waterwheels, (vi) Old shadouf, (vii)
Agriculture drainage structures, (viii) Any site or structure functionally related to present or past
agricultural water management activity.
o should be functional, achieving food security and have archival value.
 Each site is evaluated based on its merits first by the state government. The proposal is then sent to the
Centre and a team from CWC carries out an on-ground survey to verify details. Findings are presented
before the national committee, which finally submits the proposal in the prescribed format (with Central
government inputs and on-field photographs) to the international jury.
 Indian sites in the WHIS list
o In 2020, four sites selected in the list are – a) Cumbum Tank Andhra Pradesh b) KC Canal (Kurnool –
Cuddapah Canal) Andhra Pradesh c) Porumamilla Tank (Andhra Pradesh and d) Dhamapur Lake
Maharashtra.

8 www.visionias.in ©Vision IAS


o In 2018, Pedda Cheru Tank in Kamareddy district and Sadarmat Anicut in Nirmal district, both in
Telangana, were named as WHIS sites.

Q 21.C
 Aqua regia, (Latin for 'royal water') is a freshly prepared mixture of concentrated hydrochloric acid and
concentrated nitric acid in the ratio of 3:1. Hence statement 1 is correct.
 It can dissolve gold, even though neither of these acids can do so alone. Aqua regia is a highly corrosive,
fuming liquid. It is one of the few reagents that are able to dissolve gold and platinum. Hence statement 2
is correct.

Q 22.C
 Geosmin is a terpene made by microorganisms in the soil, particularly the streptomyces family of
bacteria that live in soil and decaying matter and produce most of our antibiotics.
 Geosmin is the scent of a freshly ploughed field, the first rain after a dry period, damp and
deceptively powerful. Geosmin doesn‘t seem particularly strong when you sniff the 1% solution, but
even a drop or two will have your neighbours wondering whether it‘s raining.
 Now, new research has uncovered why it's not just us humans who are attracted to this incredibly pleasant
odour. Streptomyces releases geosmin when they die, and that humans and other creatures are particularly
attuned to it.
 Springtails are big fans of geosmin. They can sense it with their antennae, are attracted to it, and will feed
on the Streptomyces producing it. Hence the correct option is (c)

Q 23.A
 Reverse Osmosis is a technology that is used to remove a large majority of contaminants from water by
pushing the water under pressure through a semi-permeable membrane.
 Total Dissolved Solids (TDS) in water are some organic and inorganic materials, which include minerals
and ions that are dissolved in a particular quantity in water. When water passes through stones, pipes or
different surfaces, the particles are absorbed into the water. TDS in water can come from different sources
such as minerals in chemicals used for treating water, runoff from the road salts and chemicals or
fertilizers from the farms.
 Water that has a TDS level of more than 1000mg/L is unfit for consumption. A high level of TDS in water
can lead to a number of health problems. The presence of potassium, sodium, chlorides increases the
TDS level in the water. However, the presence of toxic ions such as lead, nitrate, cadmium, and
arsenic present in water can lead to a number of serious health problems. This is especially important for
children because they are much more sensitive to contaminants because their defence systems have not
fully developed. Hence statement 1 is correct.
 Several types of devices are used to improve the aesthetic qualities of drinking water and to remove
chemicals. Activated carbon filters are the most common, they are more effective in removing organic
chemicals. They are often used to improve taste, smell and appearance. Hence statement 2 is not
correct.
 The desalinated water that is demineralized or deionized, is called permeate (or product) water. The
water stream that carries the concentrated contaminants that did not pass through the RO membrane is
called the reject (or concentrate) stream.

Q 24.B
 Recent Context - A team of archaeologists has discovered a cave painting in the Sulawesi Island of
Indonesia. It may be the world‘s oldest known animal cave painting dating back to more than 45,000
years. Hence option (b) is the correct answer.
 The cave painting depicts a wild boar endemic to the Sulawesi Island of Indonesia. It depicts a pig with a
short crest of upright hairs and a pair of horn-like facial warts in front of the eyes, who is likely observing
a social interaction or fight between two other warty pigs.
 Significance of this cave painting - The archaeologists noted that the dated painting of the Sulawesi
warty pig seems to be the world‘s oldest surviving representational image of an animal. These pigs have
been hunted by humans for tens of thousands of years and are the most commonly depicted animal in the
ice age rock art of the island. It suggests that they have long been used as food and form a ―focus of
creative thinking and artistic expression‖ for people of that time.
 Sulawesi Island - The island is located in central Indonesia. It is situated between Asia and Australia and
has a long history of human occupation. In 2019, a cave wall depiction of a pig and buffalo hunt became
9 www.visionias.in ©Vision IAS
the world‘s oldest recorded story. This rock painting was also found in Sulawesi. The book, ―The
Archaeology of Sulawesi‖, published by the Australian National University press in 2018 mentions that
the Sulawesi island contains some of the oldest directly dated rock art in the world and also some of the
oldest evidence for the presence of hominins beyond the southeastern limits of the Ice Age Asian
continent.

Q 25.C
 Statement 1 is not correct: Homo indicates the genus of human beings.
 Statement 2 is correct: Human beings belong to the species sapiens.
 Statement 3 is correct: All human beings belong to kingdom Animalia. All animals belonging to various
phyla are assigned to the highest category called Kingdom Animalia in the classification system of
animals.

Q 26.A
 Recent Context – The two largest planets in our solar system – Jupiter and Saturn – were seen very
close to each other (conjunction) from the earth on 21st December 2020. It occurred after nearly 400
years. The last Great Conjunction happened in 1623.
 A conjunction is the name given to any event where planets or asteroids appear to be very close together
in the sky when viewed from the Earth. Astronomers use the term Great Conjunction to describe
meetings of the two biggest planets in the solar system, Jupiter and Saturn. It is also popularly referred as
‗Christmas Star‘.
 The Great Conjunction happens once in every 20 years because of the time each of the planets take to
orbit around the Sun. Jupiter and Saturn are bright planets and can be typically seen with the naked eye
even from cities. But during a Great Conjunction, they appear to be close to each other, which is what
makes the event noteworthy.
 However, this conjunction is special as Jupiter and Saturn are a tenth of degree apart when viewed from
earth (appear as converging into one). Further, this year, the alignment of Saturn and Jupiter will occur at
night, which has not happened in over 800 years. It is because of the timing of this alignment that viewers
from nearly all over the world can expect to see this event.
 Hence option (a) is the correct answer.

Q 27.D
 Lamarckism (or Lamarckian inheritance) is the idea that an organism can pass one characteristic that it
acquired during its lifetime to its offspring (also known as the inheritance of acquired character). Hence,
statement 1 is correct.
 Lamarck incorporated the following two ideas into his theory of evolution:
o Use and disuse of organs- Individuals lose characteristics which they do not require (or use) and
develop characteristics which they do not require (or use) and develop characteristics that are
useful. Hence, statement 2 is correct.
o Inheritance acquired traits- Individuals inherit the traits of their ancestors.
 Darwin's theory (Natural selection) has been supported by a lot of evidence. Lamarck's Theory of
Inheritance of Acquired Characteristics has been disproved. This was done in two major ways. The first is
by experiment. We have seen through many real examples and observations that changes that occur in an
animal during life are not passed on to the animal's offspring. If a dog's ears are cropped short, its puppies
are stillborn with long ears. If someone exercises every day, runs marathons, eats well, and is generally
very healthy, the fitness is not passed on and the person's children still have to work just as hard to get that
fit and healthy. These and other examples show that Lamarck's theory does not explain how life formed
and became the way it is. Hence, statement 3 is not correct.

Q 28.C
 Recent Context – United Nations Environment Program (UNEP) recently released Emissions Gap Report
2020
 Emissions Gap report is annual flagship report from UNEP. It provides a yearly review of the difference
between where greenhouse emissions are predicted to be in 2030 and where they should be to avoid the
worst impacts of climate change. Hence option (c) is the correct answer.
 Emissions Gap Report 2020
o It assesses the gap between anticipated emission levels in 2030 and levels consistent with a 2°C/1.5°C
temperature target.
10 www.visionias.in ©Vision IAS
o It noted that despite a dip in 2020 carbon dioxide emissions due to Covid-19, the world is still heading
for a temperature rise in excess of 3 degree Celcius, by the end of this century. The year 2020 is on
course to be one of the warmest on record, while wildfires, storms and droughts continue to wreak
havoc.
o Forest Fires are increasing Green House Gas (GHG) emission
o G20 countries account for bulk of GHG emission. The top four emitters (China, the United States of
America, EU27+UK and India) have contributed to 55 per cent of the total GHG emissions over last
decade without land use change.
o It noted that CO2 emissions could decrease by about 7 per cent in 2020 compared with 2019 emission
levels due to COVID-19, with a smaller drop expected in GHG emissions as non-CO2 is likely to be
less affected. However, atmospheric concentrations of GHGs continue to rise.
o Although CO2 emissions will decrease in 2020, the resulting atmospheric concentrations of major
GHGs (CO2, methane (CH4) and nitrous oxide (N2O)) continued to increase in both 2019 and 2020.
Sustained reductions in emissions to reach net zero CO2 are required to stabilise global warming,
while achieving net-zero GHG emissions will result in a peak then decline in global warming.‖
o It calls for a ―green pandemic recovery‖, by increasing net-zero emission commitments.

Q 29.B
 The term ‗black hole‘ was coined in the mid-1960s by American physicist John Archibald Wheeler.
 Blackhole refers to a point in space where the matter is so compressed as to create a gravity field
from which even light cannot escape.
 Black-holes were theorized by Albert Einstein in 1915. There is a region of space beyond the black hole
called the event horizon. Hence statement 2 is correct.
 This is a "point of no return", beyond which it is impossible to escape the gravitational effects of the
black hole.
 Black holes have an intense gravitational field near them because of the large amount of mass they have
which is concentrated in a very small volume. Hence statement 1 is correct.
 If one defines the density of a black hole as the ratio of its mass to its volume, then the density decreases
as the size of the black hole increases. So the ―density‖ would depend on the size of the black hole. The
―density‖ of black holes left behind by stellar death is much, much higher than that of the earth.
 Stars end their lives in two different ways:
o those will mass around the mass of the Sun will end their lives in a gentle way, becoming a
planetary nebula and leaving behind a remnant called "white dwarf".
o Stars much more massive than the Sun explode as a supernova leaving behind either a "neutron star"
or a "black hole".
 So the Sun will never burst and will not become a black hole. Hence statement 3 is not correct.

Q 30.D
 NASA with the help of Hubble Space Telescope observed the cosmic phenomena i.e., ―A massive galaxy
or cluster of galaxies, creates a gravitational field that distorts and magnifies the light from objects
behind it but in the same line of sight‖. As a result, the galaxies appear much brighter than they
actually are. This phenomena is known as gravitational lensing. Hence option (d) is the correct
answer.
 As they‘ve been highly magnified up to 50 times. Therefore, NASA announced to conducts research in
different fields of cosmology and astronomy by analysing Gravitational lensing such as
o To study the details of early galaxies which are too far away
o To study how those galaxies are forming their stars and how that star formation is distributed
across the galaxies
o Observe invisible things in the Universe
 Geotail: The Sun emits the solar wind, which is a continuous stream of charged particles Since earth has
magnetic field which obstructs the solar wind plasma. This interaction results in the formation of a
magnetic envelope around Earth. On the Earth side facing the Sun, the envelope is compressed into a
region that is approximately three to four times the Earth‘s radius. On the opposite side, the envelope is
stretched into a long tail, which extends beyond the orbit of the Moon. This long tail is known as geotail.
o ISRO‘s CLASS instrument on Chandrayaan-2, detected charged particles on the Moon surface
during the mission at the ―Geotail‖ region around the moon. As solar flare provides a rich source
of X-rays to illuminate the surface. Therefore, it will help directly in detecting the presence of key
elements like Na, Ca, Al, Si, Ti and Fe by using CLASS instrument.

11 www.visionias.in ©Vision IAS


 Astronomers announced in October 2020 that they have witnessed a star undergoing
spaghettification. That is, the star is being ripped apart as it falls into a supermassive black hole, in a
galaxy only 215 million light-years away. They didn‘t see the star up close as it was being shredded, but
they did capture the star‘s last moments, just before it was ripped apart by the black hole, in what
astronomers call a tidal disruption event. The event created a flare of light, which they said is the closest
such flare recorded to date.
o Spaghettification is sometimes called the noodle effect. In astronomy we have long heard about the
intrepid astronaut who ventured too near a black hole‘s event horizon (the point beyond which no
light can escape) and was ―stretched like spaghetti‖ by the hole‘s powerful gravity. It‘s not just that
the hole‘s gravity is strong. It‘s that – because gravity is inversely proportional to distance, and
because a black hole‘s gravity is so exceedingly strong – the pull on the falling astronaut‘s legs is
substantially greater than the pull on his or her upper torso. Thus – as the astronaut approaches the
event horizon, the point of no return (assuming they‘re falling in feet first) – the astronaut is vertically
stretched, as illustrated in the image below, until at last disappearing (again feet first) over the event
horizon. This is spaghettification.

 Astronaut falling into a black hole (schematic illustration of the spaghettification effect). It happens
because – in the intense gravity field of a black hole – the pull on the astronaut‘s feet is greater than the
pull on his or her head.
 A supernova is the explosion of a star. It is the largest explosion that takes place in space.
o Supernovas are often seen in other galaxies. But supernovas are difficult to see in our own Milky
Way galaxy because dust blocks our view. In 1604, Johannes Kepler discovered the last observed
supernova in the Milky Way.
o What Causes a Supernova? A supernova happens where there is a change in the core, or center, of a
star. A change can occur in two different ways, with both resulting in a supernova.
 The first type of supernova happens in binary star systems. Binary stars are two stars that orbit the
same point. One of the stars, a carbon-oxygen white dwarf, steals matter from its companion star.
Eventually, the white dwarf accumulates too much matter. Having too much matter causes the star
to explode, resulting in a supernova.
 The second type of supernova occurs at the end of a single star‘s lifetime. As the star runs out of
nuclear fuel, some of its mass flows into its core. Eventually, the core is so heavy that it cannot
withstand its own gravitational force. The core collapses, which results in the giant explosion of a
supernova. The sun is a single star, but it does not have enough mass to become a supernova.
o NASA scientists use different types of telescopes to look for and study supernovas. Some
telescopes are used to observe the visible light from the explosion. Others record data from the X-rays
and gamma rays that are also produced. Both NASA‘s Hubble Space Telescope and Chandra X-
ray Observatory have captured images of supernovas.
Q 31.B
 Recent Context - The Sports Ministry has approved the inclusion of four Indigenous Games to be a part
of Khelo India Youth Games 2021. The games include: Gatka, Kalaripayattu, Thang-Ta and
Mallakhamba.
12 www.visionias.in ©Vision IAS
 Gatka: It originated from the state of Punjab. Hence pair 1 is not correctly matched.
o It is traditional fighting style of the Nihang Sikh Warriors.
o It is synonymous with swords, wooden sticks, leather shields and other sharp edged weapons.
o It is believed to have originated when sixth Sikh guru Hargobind Singh adopted ‗Kirpan‘ for self
defense during Mughal era and tenth Guru Gobind Singh made it compulsory for everyone to use the
weapons for self defense.
o Since 2008, after the formation of the Gatka Federation of India (GFI), proper rules for the game were
formulated and now it is considered a sport.
 Kalaripayattu: It is an Indian martial art and fighting style that originated in modern-day Kerala.
Hence pair 2 is correctly matched.
o It came into prominence when the local warriors found their weapons ineffective against the
Portuguese ones.
o The British promulgated a ban on Kalaripayyatu in 1804 to suppress the guerilla model revolt of
Pazhassi Kerala Varma.
o Some gurus practised this art in the interior villages of Malabar and those who learned from them
such as Kottakkal Kanaran Gurukkal preserved it for posterity.
 Thang-Ta (sword and spear): It originated from the state of Manipur. Hence pair 3 is correctly
matched.
o It is Manipuri Martial Art which is dedicated to fighting skill and worship.
o The martial art (sword fighting) form is used both as self defence as well as sports.
 Mallakhamba: The word Mallakhamb is derived from the words ‗Malla‘ and ‗Khamb‘ which means
‗Gymnast‘ and ‗Pole‘.
o Originating in the 12th century, it is a traditional sport and requires the practitioner to perform
gymnastic, aerial yoga postures and wrestling grips on standing or hanging wooden pole or cane.
o Mallakhamba participants perform gravity defying acrobatics atop a long pole or hanging ropes.
o It has been well-known across India and Madhya Pradesh and Maharashtra have been the hotspots of
this sport.

Q 32.D
 The main use of litmus is to test whether a solution is acidic or basic. Light Blue litmus paper turns
red under acidic conditions and red litmus paper turns blue under basic or alkaline conditions, with the
color change occurring over the pH range 4.5–8.3 at 25 °C (77 °F). The neutral litmus paper is purple.
 Lemon Juice and Orange Juice are acidic in nature and thus they turn the Light Blue litmus paper
into the red. Vinegar is acidic. White distilled vinegar, the kind best suited for household cleaning,
typically has a pH of around 2.5. Hence the correct option is (d)
 Sodium bicarbonate, otherwise known as baking soda, has a pH level, of around 8.4, and it is
still alkaline in nature. Human blood has a normal pH range of 7.35 to 7.45. This means that blood is
naturally slightly alkaline or basic. Hence both do not have any effect on the Light Blue litmus paper.
 Litmus can also be prepared as an aqueous solution that functions similarly. Under acidic conditions, the
solution is red, and under alkaline conditions, the solution is blue.

Q 33.D
 The xylem and the phloem make up the vascular tissue of a plant and transports water, sugars, and other
important substances around a plant. What is commonly referred to as ‗sap‘ is indeed the substances that
are being transported around a plant by its xylem and phloem.
 Phloem and xylem are closely associated and are usually found right next to one another. One xylem and
one phloem are known as a ‗vascular bundle‘ and most plants have multiple vascular bundles running the
length of their leaves, stems, and roots.
 Xylem tissue is used mostly for transporting water from roots to stems and leaves but also transports other
dissolved compounds. Phloem is responsible for transporting food produced from photosynthesis from
leaves to non-photosynthesizing parts of a plant such as roots and stems.
 Phloem
o The phloem carries important sugars, organic compounds, and minerals around a plant. Sap within the
phloem simply travels by diffusion between cells and works its way from leaves down to the roots
with help from gravity. The phloem is made from cells called ‗sieve-tube members‘ and ‗companion
cells‘. Hence, statement 1 is not correct.
 Xylem

13 www.visionias.in ©Vision IAS


o The xylem is responsible for keeping a plant hydrated. Xylem sap travels upwards and has to
overcome serious gravitational forces to deliver water to a plant‘s upper extremities, especially in tall
trees. Hence, statement 2 is not correct.

Q 34.B
 ―Athlete‘s foot‖ is a disease caused by ‗Tinea pedis‘, a fungal species. It grows well in the moist skin
between toes and the infected skin may become itchy and red.

Q 35.D
 What is DNA? DNA, or deoxyribonucleic acid, is the fundamental building block for an individual's
entire genetic makeup. It is a component of virtually every cell in the human body. Further, a person's
DNA is the same in every cell. For example, the DNA in a man's blood is the same as the DNA in his skin
cells, semen, and saliva.
 DNA is a powerful tool because each person's DNA is different from every other individual's, except for
identical twins. Because of that difference, DNA collected from a crime scene can either link a suspect to
the evidence or eliminate a suspect, similar to the use of fingerprints. It also can identify a victim through
DNA from relatives, even when nobody can be found. And when evidence from one crime scene is
compared with evidence from another, those crime scenes can be linked to the same perpetrator locally,
statewide, and across the Nation.
 Where is DNA contained in the Human body?
 DNA is contained in blood, semen, skin cells, tissue, organs, muscle, brain cells, bone, teeth, hair, saliva,
mucus, perspiration, fingernails, urine, feces, etc. Hence option (d) is the correct answer.

Q 36.D
 ELISA test: An enzyme-linked immunosorbent assay, also called ELISA or EIA, is a test that detects and
measures antibodies in your blood. This test can be used to determine if you have antibodies related to
certain infectious conditions. Antibodies are proteins that your body produces in response to harmful
substances called antigens.
 An ELISA test may be used to diagnose:
o HIV, which causes AIDS
o Lyme disease
o Pernicious anemia
o Rocky Mountain spotted fever
o Rotavirus
o Squamous cell carcinoma
o Syphilis. Hence option (d) is the correct answer.
o Toxoplasmosis
o Varicella-zoster virus, which causes chickenpox and shingles
o Zika virus
 ELISA is often used as a screening tool before more in-depth tests are ordered. A doctor may suggest this
test if you‘re having signs or symptoms of the conditions above. Your doctor may also order this test if
they want to rule out any of these conditions.
 How is the test performed?
o The ELISA test involves taking a sample of your blood. First, a healthcare provider will cleanse your
arm with an antiseptic. Then, a tourniquet, or band, will be applied around your arm to create pressure
14 www.visionias.in ©Vision IAS
and cause your veins to swell with blood. Next, a needle will be placed in one of your veins to draw a
small sample of blood. When enough blood has been collected, the needle will be removed and a
small bandage will be placed on your arm where the needle was. You‘ll be asked to maintain pressure
at the site where the needle was inserted for a few minutes to reduce blood flow.
 HIV
o The human immunodeficiency virus (HIV) targets cells of the immune system, called CD4 cells,
which help the body respond to infection. Within the CD4 cell, HIV replicates and in turn, damages
and destroys the cell. Without effective treatment of a combination of antiretroviral (ARV) drugs, the
immune system will become weakened to the point that it can no longer fight infection and disease.
 Pernicious anemia: Anemia is a medical condition in which the blood is low in normal red blood cells.
Pernicious anemia is one cause of vitamin B-12 deficiency anemia. It‘s thought mainly to be caused by
an autoimmune process that makes a person unable to produce a substance in the stomach called intrinsic
factor. This substance is needed to absorb dietary vitamin B-12 in the small intestine. This type of anemia
is called ―pernicious‖ because it was once considered a deadly disease. This was due to the lack of
available treatment.
 Syphilis: Syphilis is a chronic infectious disease caused by the spirochaete Treponema
pallidum. Syphilis is usually transmitted by sexual contact or from mother to infant, although endemic
syphilis is transmitted by non-sexual contact in communities living under poor hygiene conditions.
 Rotavirus
o Rotaviruses are the most common cause of severe diarrhoeal disease in young children throughout the
world.
o Although rotavirus infections are unpleasant, you can usually treat this infection at home with extra
fluids to prevent dehydration. Occasionally, severe dehydration requires intravenous fluids in the
hospital. Dehydration is a serious complication of rotavirus and a major cause of childhood deaths in
developing countries.
o Good hygiene, such as washing your hands regularly, is important. But vaccination is the best way to
prevent rotavirus infection.
o Symptoms A rotavirus infection usually starts within two days of exposure to the virus. Initial
symptoms are a fever and vomiting, followed by three to eight days of watery diarrhea. The
infection can cause abdominal pain as well.
 Lyme Borreliosis (Lyme disease): The spirochaete Borrelia burgdorferi, of which there are several
different serotypes.
o Infection occurs through the bite of infected ticks, both adults and nymphs, of the genus Ixodes.
o The disease usually has its onset in summer. Early skin lesions have an expanding ring form, often
with a central clear zone. Fever, chills, myalgia and headache are common. Meningeal involvement
may follow. Central nervous system and other complications may occur weeks or months after the
onset of illness. Arthritis may develop up to 2 years after onset.

Q 37.A
 Recent Context - NCAVES India Forum 2021 is being organised by the Ministry of Statistics and
Programme Implementation (MoSPI).
 Natural Capital Accounting and Valuation of the Ecosystem Services (NCAVES): It has been jointly
launched by the United Nations Statistics Division, the United Nations Environment Programme, the
Secretariat of the Convention on Biological Diversity, and the European Union in 2017. Hence statement
1 is correct.
 It aims to mainstream biodiversity and ecosystems into policy through natural capital accounts in five
participating partnership countries – Brazil, China, India, Mexico and South Africa. It aims to assist
these five participating partner countries to advance the knowledge agenda on environmental-economic
accounting, in particular ecosystem accounting. Hence statement 2 is correct.
 It will initiate pilot testing of SEEA Experimental Ecosystem Accounting (SEEA EEA) with a view to:
o Improving the measurement of ecosystems and their services (both in physical and monetary terms) at
the (sub)national level;
o Mainstreaming biodiversity and ecosystems at (sub)national level policy planning and
implementation;
o Contribute to the development of internationally agreed methodology and its use in partner countries
 The project is funded by the European Union through its Partnership Instrument (PI). Hence statement 3
is not correct.

15 www.visionias.in ©Vision IAS


 India and NCAVES
o In India, the NCAVES project is being implemented by the Ministry of Statistics and Program
Implementation (MoSPI) in close collaboration with the Ministry of Environment, Forest and Climate
Change (MoEF&CC) and the National Remote Sensing Centre (NRSC). MoSPI has coordinated with
all the stakeholders through a consultative process by setting in place a mechanism for linking the
diverse stakeholders concerned – namely producers and the policymakers using the environmental
accounts.
o The participation in the project has helped MOSPI commence the compilation of the Environment
Accounts as per the UN-SEEA framework and release environmental accounts in its publication
―EnviStats India‖ on an annual basis since 2018
o Another feather to the cap under the NCAVES project is the development of the India-EVL Tool
which is essentially a look-up tool giving a snapshot of the values of various ecosystem services in the
different states of the country,

Q 38.C
 The rusting of iron can be prevented by painting, oiling, greasing, galvanizing, chrome plating,
anodizing or making alloys.
 Galvanization is a method of protecting steel and iron from rusting by coating them with a thin layer of
zinc. The galvanized article is protected against rusting even if the zinc coating is broken.
 Alloying is a very good method of improving the properties of a metal. We can get the desired properties
by this method.
 Chrome plating (less commonly chromium plating), often referred to simply as chrome, is a technique of
electroplating a thin layer of chromium onto a metal object. The chromed layer can be decorative, provide
corrosion resistance, ease cleaning procedures, or increase surface hardness.
 Anodising is an electrolytic process for producing thick oxide coatings, usually on aluminium and its
alloys. The oxide layer is typically 5 to 30µm in thickness and is used to give improved surface resistance
to wear and corrosion, or as a decorative layer. Hence the correct option is (c)

Q 39.D
 India has consciously proceeded to explore the possibility of tapping nuclear energy for the purpose of
power generation. As result, India has planned three stage nuclear programme.
o STAGE 1 : Pressurised Heavy Water Reactor (PHWR)
o Pressurised Heavy Water Reactor using Natural UO2 as fuel matrix to produce electricity while
generating plutonium-239 as by-product.
 Heavy water as moderator and coolant
 India achieved complete self- reliance in this technology and this stage of the programme is in the
industrial domain
o STAGE 2 (FBR): Fast Breeder Reactor is India‘s second stage of nuclear power generation.
 FBR is a nuclear reactor that uses fast neutron to generate more nuclear fuels than they
consume while generating power.
 In the second stage, it envisages the use of Pu-239 obtained from the first stage reactor
operation
 As the fuel core in FBR, a blanket of U-238 surrounding the fuel core will undergo nuclear
transmutation to produce fresh Pu239 as more and more Pu-239 is consumed during the
operation.
 Besides a blanket of Th-232 around the FBR core also undergoes neutron capture reactions
leading to the formation of U-233.
 U-233 is the nuclear reactor fuel for the third stage of India‘s Nuclear Power Programme.
 FBR is known as fast breeder because it doesn‘t use moderator while liquid sodium is used
as coolant. India‘s first Prototype Fast Breeder Reactor is a 500 MWe fast breeder nuclear reactor
presently being constructed at the Madras Atomic Power Station in Kalpakkam (Tamil Nadu).
o STAGE 3: (Breeder Reactor). A Stage III reactor or an Advanced nuclear power system involves a
self-sustaining series of thorium-232-uranium-233 fuelled reactors.
 U-233, obtained from second stage will be used as fuel in nuclear reactor.
 Besides, U-233 fuelled breeder reactors will have a Th-232 blanket around the U-233 reactor core
which will generate more U-233 as the reactor goes operational thus resulting in the production of
more and more U-233 fuel from the Th-232 blanket as more of the U-233 in the fuel core is
consumed helping to sustain the long term power generation fuel requirement.
 Hence all the statements are correct.
16 www.visionias.in ©Vision IAS
Q 40.C
 Nuclear fusion is a reaction in which two or more atomic nuclei are combined to form one or more
different atomic nuclei and subatomic particles (neutrons or protons).
 The main fuels used in nuclear fusion are deuterium and tritium, both heavy isotopes of hydrogen.
Deuterium constitutes a tiny fraction of natural hydrogen, only 0,0153%, and can be extracted
inexpensively from seawater. Tritium can be made from lithium, which is also abundant in nature.
 Helium-3 (He3) is a gas that has the potential to be used as a fuel in future nuclear fusion power plants.
There is very little helium-3 available on the Earth. However, there are thought to be significant supplies
on the Moon.
 Zirconium is a lustrous, grey-white, strong transition metal that closely resembles hafnium and, to a
lesser extent, titanium. It is not a nuclear fusion fuel.

Q 41.A
 Electric Fuse is a current interrupting device that protects the electrical circuit.
 The principle behind the operation of the fuse is the heating effect of the electrical current. If the
current passes through a conductor with a certain resistance, the loss due to the conductor's resistance is
dissipated in the form of heat. Under normal operating conditions, the heat produced in the fuse element is
easily dissipated into the environment due to the flow of current through it. As a result, the fuse part
remains at a temperature below its melting point. Hence statement 1 is correct.
 Whenever some faults, such as short circuits, occur, the current flow through the fuse element exceeds the
prescribed limits. This creates an excess of heat that melts the fuse part and breaks the circuit.
 So if the electrical current passing through the fuse goes beyond what the device can handle, the
fuse link is melted and the circuit is opened to prevent damage to the electrical component.
 Fuses are always connected in series with the component to be protected from overcurrent, so that when
the fuse blows (opens) it will open the entire circuit and stop current through the component. Hence
statement 2 is not correct.

Q 42.D
 Each organism is adapted to its environment. The form of nutrition differs depending on the type and
availability of food material as well as how it is obtained by the organism.
 Chemoheterotrophs are unable to utilize carbon dioxide to form their own organic compounds. Their
carbon source is rather derived from sulfur, carbohydrates, lipids, and proteins. Chemoheterotrophs are
only able to thrive in environments that are capable of sustaining other forms of life due to their
dependence on these organisms for carbon sources. Chemoheterotrophs are the most abundant type of
chemotrophic organisms and include most bacteria, fungi and protozoa.
 Yeast, mushroom and bread mould have a saprophytic mode of nutrition which is chemoheterotrophic in
nature. They break down complex organic substances by secreting digestive enzymes outside their body
and absorb simple molecules as nutrients. Hence option (d) is the correct answer.
 Phototrophs are organisms that carry out photon capture to acquire energy. Photoautotrophs convert
inorganic materials into organic materials for use in cellular functions such as biosynthesis and respiration
and provide nutrition for many other forms of life. Photoheterotrophs depend on light for their source of
energy and mostly organic compounds from the environment for their source of carbon. i.e.
cyanobacteria- that build up carbon dioxide and water into organic cell materials using energy from
sunlight, starch is produced as the final product.

Q 43.B
 Proton, neutron, and electron are tiny particles that makeup atoms. The neutrino is also a tiny
elementary particle, but it is not part of the atom. Such particles are also found to exist in
nature. Hence statement 1 is correct.
 Neutrino has a very tiny mass and no charge. It interacts very weakly with other matter particles. So
weakly that every second trillion of neutrinos fall on us and pass through our bodies unnoticed.
 Neutrinos come from the sun (solar neutrinos) and other stars, cosmic rays that come from beyond the
solar system, and from the Big Bang from which our Universe originated. They can also be produced in
the lab. Hence statement 3 is correct.
 The INO (India based Neutrino Observatory) will study atmospheric neutrinos only. Solar neutrinos
have much lower energy than the detector can detect. INO Project is aimed at building a world-class
underground laboratory with a rock cover to conduct basic research on the neutrino.

17 www.visionias.in ©Vision IAS


 The Tata Institute of Fundamental Research is the nodal institution. The observatory is to be built
jointly with the Department of Atomic Energy and the Department of Science and Technology. The
observatory will be located underground so as to provide adequate shielding to the neutrino detector from
cosmic background radiation.
 The operation of INO will have no release of radioactive or toxic substances. It is not a weapons
laboratory and will have no strategic or defence applications.
 They are the second most abundant particle in the universe after particles of light called photons. Hence
statement 2 is not correct.

Q 44.D
 Epigenetics is the study of heritable changes in gene expression (active versus inactive genes) that do
not involve changes to the underlying DNA sequence (i.e. not encoded in DNA sequence) — a
change in phenotype without a change in genotype — which in turn affects how cells read the genes.
Hence statement 1 is not correct.
 Epigenetic change is a regular and natural occurrence but can also be influenced by several factors
including age, the environment/lifestyle, and disease state.
 Unlike genetic changes, epigenetic changes are reversible and do not change the DNA sequence, but
they can change how the body reads a DNA sequence. Hence statement 2 is not correct.
 Epigenetic modifications can manifest as commonly as the manner in which cells terminally differentiate
to end up as skin cells, liver cells, brain cells, etc. Or, epigenetic change can have more damaging effects
that can result in diseases like cancer.
 At least three systems including DNA methylation, histone modification, and non-coding RNA (ncRNA)-
associated gene silencing are currently considered to initiate and sustain epigenetic change.
 Encouraging evidence has linked epigenetic effects to
o oncogenesis, progression, and treatment of cancer, Hence statement 2 is correct.
o the regulation of development and function of the nervous system,
o gene regulation,
o cellular stress events,
o nutrigenomics,
o aging and DNA repair

Q 45.D
 DNA fingerprinting is a chemical test that shows the genetic makeup of a person or other living things.
It‘s used as evidence in courts, to identify bodies, track down blood relatives, and to look for cures for
disease.
 Uses:
 DNA fingerprinting most often has been used in court cases and legal matters. It can:
o Physically connect a piece of evidence to a person or rule out someone as a suspect.
o Show who your parents, siblings, and other relatives may be.
o Identify a dead body that‘s too old or damaged to be recognizable.
o DNA fingerprinting is extremely accurate. Most countries now keep DNA records on file in much the
same way police keep copies of actual fingerprints.
o For criminal identification. Hence, option1 is correct.
o To resolve disputes of maternity/paternity. Hence, option 2 is correct.
o To identify mutilated remains. Hence, option 3 is correct.
o In cases of exchange of babies in a hospital ward. Hence, option 4 is correct.
o In forensic wildlife. Hence, option 5 is correct.
 It also has medical uses. It can:
 Match tissues of organ donors with those of people who need transplants.
 Identify diseases that are passed down through your family.
 Help find cures for those diseases, called hereditary conditions.

Q 46.B
 What is trans fat?
o Trans fat, or trans-fatty acids, are unsaturated fatty acids that come from either natural or industrial
sources. Naturally-occurring trans fat comes from ruminants (cows and sheep). Industrially-produced
trans fat is formed in an industrial process that adds hydrogen to vegetable oil converting the liquid
into a solid, resulting in ―partially hydrogenated‖ oil (PHO). Hence, statement 1 is not correct.

18 www.visionias.in ©Vision IAS


 What is the health impact of consuming foods that contain trans fat?
o Approximately 540,000 deaths each year can be attributed to the intake of industrially-produced trans-
fatty acids.1 High trans fat intake increases the risk of death from any cause by 34%, coronary heart
disease deaths by 28%, and coronary heart disease by 21%.2 This is likely due to the effect on lipid
levels: trans fat increases LDL (―bad‖) cholesterol levels while lowering HDL (―good‖) cholesterol
levels. Trans fat has no known health benefits. Hence, statement 3 is correct.
 Why are partially hydrogenated oils used in food?
o Partially hydrogenated oils (PHO) are solid at room temperature and prolong the shelf life of
products. They are primarily used for deep frying and as an ingredient in baked goods. PHOs were
first introduced into the food supply in the early 20th century as a replacement for butter and lard;
they are not a natural part of the human diet and are fully replaceable.
 Which foods contain industrially-produced trans fat?
o Partially hydrogenated oils (PHO) are the main source of industrially-produced trans fat. PHO is an
ingredient in many foods, including margarine, vegetable shortening, and Vanaspati ghee; fried foods
and doughnuts; baked goods such as crackers, biscuits, and pies; and pre-mixed products such as
pancake and hot chocolate mix. Baked and fried street and restaurant foods often contain industrially-
produced trans fat. All of these products can be made without industrially-produced trans fat. Hence,
statement 2 is correct.

Q 47.A
 A fluorescent lamp, or fluorescent tube, is a low-pressure mercury-vapour gas-discharge lamp that
uses fluorescence to produce visible light.
 An electric current in the gas excites mercury vapour, which produces short-wave ultraviolet light that
then causes a phosphor coating on the inside of the lamp to glow. Hence option (a) is the correct
answer.
 Astatine is a radioactive element of the halogen series: a decay product of uranium and thorium that
occurs naturally in minute amounts and is artificially produced by bombarding bismuth with alpha
particles. There are currently no uses for astatine outside of research.
 Cinnabar is a toxic mercury sulfide mineral with a chemical composition of HgS. It is the only important
ore of mercury. It has a bright red colour that has caused people to use it as a pigment, and carve it into
jewellery and ornaments for thousands of years in many parts of the world.

Q 48.C
 A geostationary orbit or geosynchronous equatorial orbit is a circular geosynchronous orbit above
Earth's equator and following the direction of Earth's rotation.
 Geostationary orbit can receive signals with a simple antenna as the satellite is in the relatively same
position (DTH, VSAT services). The parabolic antenna is used to nullify the effect of atmospheric
distortions). Hence statement 2 is not correct.
 The geostationary orbit is disadvantaged for certain types of services such as voice calls and video
conferencing because of the relatively large time delay (about 0.25 sec) experienced by a
communication signal in travelling from the earth to a geostationary satellite and back. For this reason,
GEO systems are best utilized where the time delay is not an issue as is the case with direct TV
service and Internet data services.
 Such satellites orbit once a day and move in the same direction as the Earth. So, such
satellites appear stationary above the same point on the earth‘s surface. They can only be above the
Equator and are placed at an altitude of 35,790 km. Hence statement 1 is correct.
 Geostationary orbit is at an altitude very close to 35,786 km and which keeps the satellite fixed over one
longitude at the equator. Thus there is only one Geostationary orbit. Scientists have alerted the world
about possible dangerous collision events as the satellites are crowding at Earth‘s geostationary
orbit. Hence statement 3 is correct.
Q 49.D
 Bacteria are microscopic, single-cell organisms that live on and inside plants, animals, and people. Some
bacteria can infect the body of human or Plant that led to the bacterial disease. Some of the bacterial
diseases are:
o Diphtheria: It is an infection caused by the bacterium Corynebacterium diphtheriae. Hence option 1
is correct.
o Tetanus: It is an infection caused by bacteria called Clostridium tetani. Hence option 2 is correct.

19 www.visionias.in ©Vision IAS


o Leprosy is an infectious disease caused by a bacillus, Mycobacterium leprae. The disease mainly
affects the skin, the peripheral nerves, mucosa of the upper respiratory tract, and the eyes. It is curable
with multidrug therapy (MDT). It is likely transmitted via droplets, from the nose and mouth, during
close and frequent contact with untreated cases. India officially eliminated leprosy in 2005, reducing
its prevalence rate to 0.72 per 10,000 people at national level but still hosts highest patients of leprosy
in the world. Hence option 3 is correct.
o Whooping cough, also called pertussis, is a serious respiratory infection caused by a type of bacteria
called Bordetella pertussis. It can cause serious illness in babies, children, teens, and adults. Some
other diseases which are caused by bacteria are: Cholera, typhoid, tuberculosis. Plague, and Anthrax
etc. Hence option 4 is correct.
Q 50.B
 Steady advances in voltage and applications have made cordless tools the first choice on most projects.
For years NiCad (Nickel Cadmium) batteries reigned supreme. Recently, though, they‘ve been displaced
by newer Lithium-Ion battery technology.
 Lithium-ion batteries are smaller and lighter than a NiCad battery. Lithium-ion also two to three
times more expensive than NiCad. On the other hand, Lithium-ion has virtually no self-discharge.
This allows a lithium-ion battery to be stored for months without losing charge. Thus require less
maintenance and are environmentally safer than Nickel-cadmium (NiCad) batteries. Hence the
correct option is (b)
 Nickel-cadmium batteries suffer from a ―memory effect.‖ The battery remembers the point in their
charge cycle where recharging began. During subsequent uses, the voltage will drop at that point as if it
had been discharged. That‘s why it‘s wise that you use a NiCad battery until it‘s totally dead before
recharging with this kind of proper use, a nickel-cadmium battery can last for 1,000+ cycles before losing
capacity.
 Lithium-ion batteries, on the other hand, are low maintenance. They resist the ―memory effect‖ and
tolerate a wider range of temperatures. Their only serious drawback is fragility. They also require a
protection circuit to keep working safely.
 Both will perform for roughly the same number of cycles. While the Lithium-ion battery may provide
more voltage, it does so at a significantly higher cost compared to a NiCad battery.
 Most lithium-ion battery fires and explosions come down to a problem of short-circuiting. This
happens when the plastic separator fails and lets the anode and cathode touch. And once those two get
together, the battery starts to overheat.
o Batteries left too close to a heat source—or caught in a fire—have been known to explode. Other
external factors can cause a lithium-ion battery to fail, too. If you drop your phone too hard (or too
many times), there‘s a chance you‘ll damage the separator and cause the electrodes to touch. If you
pierce the battery (either by accident or deliberately), then you‘ll almost certainly cause a short circuit.
 NiCd batteries also produce hydrogen or oxygen gas if overcharged, causing the possibility of rupture or
explosion.
 Since absolute performance is roughly equal, choosing between Lithium-ion and NiCad batteries
comes down to simple differences: Lighter, longer-lasting and more expensive, or heavier, more
consumable and cheaper.
Q 51.C
 A bacteriophage is a type of virus that infects bacteria. The word "bacteriophage" literally means
"bacteria eater," because bacteriophages destroy their host cells. All bacteriophages are composed of a
nucleic acid molecule that is surrounded by a protein structure. A bacteriophage attaches itself to a
susceptible bacterium and infects the host cell. Hence option (c) is the correct answer.
 Following infection, the bacteriophage hijacks the bacterium's cellular machinery to prevent it from
producing bacterial components and instead forces the cell to produce viral components. Eventually, new
bacteriophages assemble and burst out of the bacterium in a process called lysis. Bacteriophages
occasionally remove a portion of their host cells' bacterial DNA during the infection process and then
transfer this DNA into the genome of new host cells. This process is known as transduction.
 Since their discovery, bacteriophages have been considered to be potential antibacterial therapeutics
for the treatment of various infectious diseases in humans. In recent times, the rapid rise of multi-drug
resistant bacteria worldwide has led to a renewed interest in phage therapy as a possible alternative to
antibiotics or, at least, a supplementary approach for the treatment of some bacterial infections.
Furthermore, bacteriophage-based vaccination is emerging as one of the most promising preventive
strategies.

20 www.visionias.in ©Vision IAS


Q 52.D
 Recent Context – The GGW countries, according to a United Nations report released September 2020,
need to speed up the current pace of land restoration to an average of 8.2 million hectares every year. And
France has rushed to help: President Emmanuel Macron January 11, 2021 announced $14 billion to scale
up work.
 Great Green Wall (GGW) Initiative: It was started in 2007.
o It is an African led initiative to reverse land degradation and desertification running from Senegal
(West) to Djibouti (East). Hence statement 1 is not correct.
o The objective of the initiative is to grow an 8,000km-long line of trees and plants across the entire
Sahel - Senegal (West) to Djibouti (East). By 2030, the ambition of the initiative is to restore 100
million ha of currently degraded land; sequester 250 million tons of carbon and create 10 million
green jobs.
o The Great Green Wall is now being implemented in more than 20 countries across Africa and more
than eight billion dollars have been mobilized and pledged for its support. The 11 countries selected
as intervention zones for the Great Green Wall are: Burkina Faso, Chad, Djibouti, Eritrea, Ethiopia,
Mali, Mauritania, Niger, Nigeria, Senegal, and Sudan.
o The United Nations to Combat desertification is a key partner in this initiative. The project is not
funded by Green Climate Fund. Hence statement 2 is not correct.
o Once complete, the Great Green Wall will be the largest living structure on the planet, 3 times the size
of the Great Barrier Reef.

Q 53.B
 Isotopes are defined as the atoms of the same element, having the same atomic number but different
mass numbers. Hydrogen atom has three atomic species, namely protium (1 1H), deuterium ( 2 1H
or D) and tritium ( 3 1H or T). The atomic number of each one is 1, but the mass number is 1, 2 and
3, respectively. Hence option (b) is the correct answer.
 While Isobar is defined as , atoms with identical atomic number but different atomic mass number for e.g.
two elements — calcium, atomic number 20, and argon, atomic number 18 but the mass number of both
these elements is 40.

Q 54.A
 Indian Agricultural Research Institute has developed 'decomposer' capsules, which when mixed in a
water solution and sprayed on land, gets to work on paddy stubble, softening and decomposing it to the
extent it can mix with soil and act as compost.
 It is essentially a fungi-based liquid solution that can soften hard stubble to the extent that it can be
easily mixed with soil in the field to act as compost.
 This would then rule out the need to burn the stubble, and also help in retaining the essential microbes and
nutrients in the soil that are otherwise damaged when the residue is burned.
 It has been specifically developed to deal with the problem of stubble burning by softening the hard
stubble. Hence option (a) is the correct answer.
 Incineration is a waste treatment process that involves the combustion of organic substances contained in
waste materials.
 Bioremediation is a branch of biotechnology that employs the use of living organisms, like microbes and
bacteria, in the removal of contaminants, pollutants, and toxins from soil, water, and other environments.

Q 55.B
 Recent Context - A two-day water bird census was conducted in Andhra Pradesh as part of Asian
Waterbird census – 2021. It covered at least two dozen sites, including Coringa Wildlife Sanctuary,
Kolleru Lake and Krishna Sanctuary.
 Asian Waterbird census: It is a citizen-science programme supporting conservation and management of
wetlands and waterbirds worldwide.
 It is an annual event and takes place during the second and third weeks of January. Hence
statement 1 is not correct.
 The census has six major objectives:
o providing the basis for estimates of waterbird populations;
o monitoring changes in waterbird numbers and distribution by regular, standardised counts of
representative wetlands;
o improving knowledge of little-known waterbird species and wetland sites;
21 www.visionias.in ©Vision IAS
o identifying and monitoring (networks of) sites that are important for waterbirds in general and, more
specifically, identifying and monitoring sites that qualify as Wetlands of International Importance
under the Ramsar Convention on Wetlands;
o providing information on the conservation status of waterbird species, for use by international
agreements;
o increasing awareness of the importance of waterbirds and their wetland habitats at local, national and
international levels.
 The AWC also aims to build and strengthen national networks of enthusiastic volunteers and facilitate
their training.
 In India, the AWC is jointly coordinated by the Bombay Natural History Society and Wetlands
International. The census is primarily carried out by volunteers from all walks of life: university and
school staff and students, nature club members, amateur and professional ornithologists, government and
non-governmental agencies related to forestry, wildlife or wetland conservation and others.
 The census was initiated in 1987 in the Indian subcontinent, and has grown rapidly to cover 26 countries
of South Asia, East Asia, South East Asia, Pacific, as well as Russia Far East.
 The AWC runs parallel to other international census of waterbirds in Africa, Europe and Neotropics under
the umbrella of the International Waterbird Census (IWC).
 Sites covered during the census include all types of natural and man-made wetlands, including,
rivers, lakes, reservoirs, ponds, freshwater swamps, mangroves, mudflats, coral reefs, rice fields and
sewage farms, etc. Hence statement 2 is correct.
 Waterbirds counted during the census include all types of waterbirds regularly encountered at wetlands,
including, grebes, cormorants, pelicans, herons, egrets, etc. In addition, raptors, kingfishers and other
wetland dependent birds are covered. Count data is entered onto standardised count forms.
 The data collected by the AWC has been featured in various reports and have contributed to conservation
activities ranging from local to global levels.

Q 56.B
 Free-space optical communication (FSO) is an optical communication technology that uses light
propagating in free space to wirelessly transmit data for telecommunication or computer networking.
"Free-space" means air, outer space, vacuum, or something similar. Hence statement 1 is correct.
 FSO is a line-of-sight technology that uses lasers to provide optical bandwidth connections. Currently,
FSO is capable of up to 2.5 Gbps of data, voice and video communications through the air, allowing
optical connectivity without requiring a fibre-optic cable or securing spectrum licenses. Hence statement
2 is correct.
 This contrasts with using solids such as optical fibre cable. FSO communications can provide high data
rates in Gbits/s ranges through the atmosphere for ranges from a few hundreds of meters to a few
kilometres.
 FSO links include the following:
o chip-to-chip communication,
o indoor infrared (IR) or VLC,
o interbuilding communication, and
o free-space laser communications including airborne, spaceborne, and deep space missions.
 Free-space point-to-point optical links can be implemented using infrared laser light, although low-data-
rate communication over short distances is possible using LEDs, Infrared Data Association (IrDA)
technology is a very simple form of free-space optical communications. Hence statement 3 is correct.
 There are several transmission windows between 780 nm and 1600 nm wavelength range. These windows
are very suitable for FSO operation due to its low attenuation, as well as the availability of high-quality
transmitter and detector components.

Q 57.C
 A cryogenic engine/ cryogenic stage is the last stage of space launch vehicles which makes use of
Cryogenics. Cryogenics is the study of the production and behaviour of materials at extremely low
temperatures (below -150 degree Centigrade) to lift and place the heavier objects in space.
 The cryogenic engine makes use of Liquid Oxygen (LOX) and Liquid Hydrogen (LH2) as
propellants which liquefy at -183 deg C and -253 deg C respectively. LOX and LH2 are stored in their
respective tanks. The major components of a cryogenic rocket engine are both its fuel and oxidizer are
gases liquefied and stored at very low temperatures. Hence statement 1 is correct.

22 www.visionias.in ©Vision IAS


 Hydrogen and oxygen are extremely clean fuels. When they combine, they give out only water. This
water is thrown out of the nozzle in form of very hot vapour. Thus the rocket is nothing but a high burning
steam engine.
 Cryogenic engines are usually prepared for the ―upper stages" i.e. the last stage of the rocket because this
stage provides 50% of the velocity of 10.2 km per second needed at the point of injection of a
satellite. Hence statement 2 is correct.

Q 58.C
 Recently, the Singapore Food Agency (SFA) approved the sale of a lab-grown meat product. This is the
first time cultured meat has been cleared for sale anywhere in the world.
 Lab-grown meat, also known as cultured meat on account of the process, is meat that is prepared
from the regeneration of cell animal cultures, instead of slaughtered animals. Hence statement 1 is
correct.
 Using techniques typically used during regenerative medicine, scientists use animal cells to ‗grow‘ more
cells by feeding them nutrients like salts and sugars in a controlled lab environment. Once enough muscle
fibers have grown, the meat begins to vaguely resemble the animal it was supposed to recreate. The
resulting ‗meat‘ can then be shaped into patties or sausages. These do resemble actual meat in terms of
appearance, texture, and even nutrient profile.
 Mock meat or vegan meat are plant proteins that are processed to resemble meat. They do not
contain any animal fat or animal proteins, unlike lab-grown meat. Mock meat doesn‘t require any animals
at all in the production process and is completely cruelty-free.
 India consumed 3.9 million metric tonnes of poultry (1 tonne = 1,000kg) in 2020 alone. Those
numbers pale in comparison to China‘s 20 million tonnes and the US consumed 19 million tonnes in
2020.
 Lab-meat will effectively eliminate the need to consume so much livestock, resulting in a 46%
reduction in energy consumption required for rearing them. Furthermore, it will also reduce water
usage. At present, 9,000 litres of water are required to produce 1kg of meat in India as compared to 94
litres required for cultured meat. It will also reduce carbon emissions by 14.5%, as feeding cattle for
good health belch up methane, a potent climate-changing gas. Hence statement 2 is correct.

Q 59.D
 Recent Context - ‗SolarWinds hack‘ is a cyberattack recently discovered in the United States of
America. It has emerged as one of the biggest ever targeted cyberattack against the US government, its
agencies and several other private companies.
 SolarWind hack is a cyberattack recently discovered in USA. It was first discovered by US cybersecurity
company FireEye. Hence option (d) is the correct answer.
 It is being called a ‗Supply Chain‘ attack where instead of directly attacking the federal government or a
private organisation‘s network, the hackers target a third-party vendor, which supplies software to them.
In this case, the target was an IT management software called Orion, supplied by the Texas-based
company SolarWinds. SolarWinds says 18,000 of its clients have been impacted.
 According to FireEye, the hackers gained access to victims via trojanized updates to SolarWinds‘ Orion
IT monitoring and management software. A software update was exploited to install the ‗Sunburst‘
malware into Orion, which was then installed by more than 17,000 customers. Once installed, the
malware gave a backdoor entry to the hackers to the systems and networks of SolarWinds‘ customers. The
malware was also able to thwart tools such as anti-virus that could detect it.
 Cyberattack - It is a malicious and deliberate attempt by an individual or organization to breach the
information system of another individual or organization. Usually, the attacker seeks some type of benefit
from disrupting the victim‘s network.
 Some of the common types of cyber attacks are:
o Malware - It is malicious software, including spyware, ransomware, viruses, and worms. Malware
breaches a network through a vulnerability, typically when a user clicks a dangerous link or email
attachment that then installs risky software.
o Phishing - It is the practice of sending fraudulent communications that appear to come from a
reputable source, usually through email. The goal is to steal sensitive data like credit card and login
information or to install malware on the victim‘s machine.
o Man-in-the-middle attack - Man-in-the-middle (MitM) attacks, also known as eavesdropping
attacks, occur when attackers insert themselves into a two-party transaction. Once the attackers
interrupt the traffic, they can filter and steal data.

23 www.visionias.in ©Vision IAS


o Zero-day exploit - A zero-day exploit hits after a network vulnerability is announced but before a
patch or solution is implemented.
o Denial of Service Attack - A denial-of-service attack floods systems, servers, or networks with
traffic to exhaust resources and bandwidth. As a result, the system is unable to fulfill legitimate
requests. Attackers can also use multiple compromised devices to launch this attack. This is known as
a distributed-denial-of-service (DDoS) attack.

Q 60.D
 Recent context – Ministry of Power, in association with Bureau of Energy Efficiency (BEE), has
launched SAATHEE (State-wise Actions on Annual Targets and Headways on Energy Efficiency) portal.
 SAATHEE (State-wise Actions on Annual Targets and Headways on Energy Efficiency) portal: It is
a Management Information System (MIS) portal which will facilitate real-time monitoring of the progress
of implementation of various Energy Conservation endeavours at State level. Hence option (d) is the
correct answer.
 It has been developed by Bureau of Energy Efficiency (BEE).
 About Bureau of Energy Efficiency (BEE): It is a statutory body under Ministry of Power.
 It is mandated to implement policy and programmes in energy efficiency and conservation. The objective
of such initiatives is to reduce energy intensity in our country by optimizing energy demand and reduce
emissions of greenhouse gases (GHG) which are responsible for global warming and climate change.

Q 61.D
 The halogens are a series of non-metal elements in the periodic table consisting of five chemically
related elements: fluorine (F), chlorine (Cl), bromine (Br), iodine (I), and astatine (At). The recently
artificially created element 117, Tennessine (Ts), may also be a halogen. In the modern IUPAC
nomenclature, this group is known as group 17.
 The name "halogen" means "salt-producing". When halogens react with metals, they produce a wide
range of salts, including calcium fluoride, sodium chloride (common table salt), silver bromide and
potassium iodide.
 The group of halogens is the only periodic table group that contains elements in three of the main states of
matter at standard temperature and pressure. All of the halogens form acids when bonded to hydrogen.
Most halogens are typically produced from minerals or salts.
 The middle halogens—chlorine, bromine, and iodine—are often used as disinfectants in the water-
purification system. Organobromides are the most important class of flame retardants, while elemental
halogens are dangerous and can be lethally toxic.
 During springtime in the polar regions, unique photochemistry converts inert halide salt ions (e.g.
Br−) into reactive halogen species (e.g. Br atoms and BrO) that deplete ozone in the boundary layer
to near-zero levels. Hence the correct option is (d)

Q 62.A
 Recent Context – The Reserve Bank of India (RBI) has released RBI - Digital Payments Index (DPI).
 RBI -Digital Payment Index (DPI)
 The Reserve Bank of India has released a Digital Payments Index to measure the growth in cashless
transactions in India over different time periods. Hence statement 1 is correct.
 The index comprises five broad parameters with varying weights to measure the penetration of digital
payments, the central bank said in a statement on Friday. The five key parameters include:
o Payment enablers (25%).
o Payment infrastructure—demand-side factors (10%).
o Payment infrastructure—supply-side factors (15%)
o Payment performance (45%).
o Consumer centricity (5%).
 These factors include multiple sub-parameters that would help the regulator conduct its study into the
digital payment ecosystem.
 Payment enablers comprises multiple channels through which digital payments can be accessed. This
includes the internet, mobile, Aadhaar, bank accounts, merchants, and participants.
 The sub-parameters for payment infrastructure include credit cards, debit cards, prepaid payment
instruments, point of sale terminals, automated teller machines, quick response codes, among others.

24 www.visionias.in ©Vision IAS


 Under payment performance, which carries the highest weight in the index, the regulator would measure
factors such as the volume and value of digital payments, unique users, paper clearing, currency in
circulation and cash withdrawals.
 For customer centricity, the RBI would consider customer education and awareness, frauds, complaints,
declines and system downtime.
 The RBI-DPI has been constructed with March 2018 as the base period. The DPI for March 2019 and
March 2020 was 153.47 and 207.84 respectively, indicating appreciable growth.
 Going forward, the index shall be published on RBI‘s website on a semi-annual basis from March
2021 with a lag of four months. Hence statement 2 is not correct.

Q 63.A
 Of the five states, the matter can be in, the Bose-Einstein condensate is perhaps the most
mysterious. Gases, liquids, solids, and plasmas were all well studied for decades, if not centuries; Bose-
Einstein condensates weren't created in the laboratory until the 1990s.
 A Bose-Einstein condensate is a group of atoms cooled to within a hair of absolute zero. When they
reach that temperature the atoms are hardly moving relative to each other; they have almost no free energy
to do so. At that point, the atoms begin to clump together and enter the same energy states. They become
identical, from a physical point of view, and the whole group starts behaving as though it were a single
atom.
 This state was first predicted, generally, in 1924–1925 by Albert Einstein following and crediting a
pioneering paper by Satyendra Nath Bose on the new field now known as quantum statistics.
 One application for BEC is for the building of so-called atom lasers, which could have applications
ranging from atomic-scale lithography to measurement and detection of gravitational fields. Hence
the correct option is (a).

Q 64.D
 Geospatial technology is an umbrella phrase associated with a range of various technologies which
include remote sensing, Global Positioning System (GPS), Geographic Information System (GIS),
information technologies, and field sensors. That are intended to facilitate the process of
capturing/storing/ processing/ displaying/ disseminating information tied to a location.
 Applications of Geospatial technology
o Natural resource Management: It can be used most effectively for environmental data analysis and
planning such as changes in land use and land cover, forest degradation, soil types, mapping soil-
erosion intensity, aquifers & ground water. For e.g. India-WRIS (Water Resources Information
System) developed by the ISRO to provide a single portal for all water resources data and
information. Hence options 1 and 3 are correct.
o Improving the health condition: It uses technology to know hospital location, settlements and
demographics, the collection and analysis of spatial patterns of disease appearance and disease-
specific deaths, taking into consideration the social, economic, ecological and demographic
prerequisites of space and time. Hence option 4 is correct.
o Disaster risk reduction and resilience: Geospatial technology helps in early warning and faster
response during the time of disaster on the basis of collected data. For e.g. Remote sensing satellites
(like OCEANSAT series, CARTOSAT series) and earth observation satellites like (Cartosat-1 & 2,
Resourcesat-1 & 2 etc.) are also being utilised for vulnerability mapping and for taking preventive
measures. Hence option 2 is correct.
o Energy: To determine the suitability of a location for generating solar energy the ISRO uses solar
data derived from measurements on-board the geostationary satellite Kalpana. o Connectivity:
Geospatial information through Road Asset Management system assist in accurate and scientific
maintenance planning, enhance road safety measures and plan the development of the National
Highways network in India.

Q 65.B
 Radioactivity is the property exhibited by certain types of matter of emitting energy and subatomic
particles spontaneously. It is, in essence, an attribute of individual atomic nuclei. An unstable nucleus
will decompose spontaneously, or decay, into a more stable configuration but will do so only in a
few specific ways by emitting certain particles or certain forms of electromagnetic energy.
 Radioactive decay is a property of several naturally occurring elements as well as of artificially produced
isotopes of the elements. The rate at which a radioactive element decays is expressed in terms of its half-
life; i.e., the time required for one-half of any given quantity of the isotope to decay.
25 www.visionias.in ©Vision IAS
 The emissions of the most common forms of spontaneous radioactive decay are the alpha (α) particle, the
beta (β) particle, the gamma (γ) ray, and the neutrino. The alpha particle is actually the nucleus of a
helium-4 atom, with two positive charges He (mass number 4, atomic number 2). Such charged atoms
are called ions. The neutral helium atom has two electrons outside its nucleus balancing these two
charges.
 Beta particles may be negatively charged (beta minus, symbol e−), or positively charged (beta plus,
symbol e+). The beta minus [β−] particle is actually an electron created in the nucleus during beta decay
without any relationship to the orbital electron cloud of the atom. The beta plus particle, also called the
positron, is the antiparticle of the electron; when brought together, two such particles will mutually
annihilate each other.
 Gamma rays are electromagnetic radiations such as radio waves, light, and X-rays. Beta
radioactivity also produces the neutrino and antineutrino, particles that have no charge and very little
mass, symbolized by ν and ν, respectively. Thus only gamma rays and neutrino are electrically neutral and
not alpha or beta particles. Hence statement 1 is not correct.
 Beta radiation is more penetrating than alpha radiation. It can pass through the skin, but it is absorbed by a
few centimetres of body tissue or a few millimetres of aluminium. Gamma radiation is the most
penetrating of the three radiations.
 Alpha particles can easily be shielded by a single sheet of paper and cannot penetrate the outer dead
layer of skin, so they pose no danger when their source is outside the human body. Beta particles are
essentially electrons emitted from the nucleus of a radioactive atom.
 Gamma rays are energy that has no mass or charge. They have tremendous penetration power and
require several inches of dense material (like lead) to shield them. Hence statement 2 is correct.

Q 66.A
 Respiration in aquatic organisms: Since the amount of dissolved oxygen is fairly low compared to
the amount of oxygen in the air, the rate of breathing in aquatic organisms is much faster than that
seen in terrestrial organisms. Fishes take in water through their mouths and force it past the gills where the
dissolved oxygen is taken up by the blood. Hence, statement 1 is correct and 2 is not correct.
 Terrestrial organisms use oxygen in the atmosphere for respiration. This oxygen is absorbed by different
organs in different animals. All these organs have a structure that increases the surface area which is in
contact with the oxygen-rich atmosphere. Since the exchange of oxygen and carbon dioxide has to take
place across this surface, this surface is very fine and delicate. In order to protect this surface, it is usually
placed within the body, so there have to be passages that will take air to this area.

Q 67.C
 Germline gene therapy is when DNA is transferred into the cells that produce reproductive cells, eggs,
or sperm, in the body. This type of therapy allows for the correction of disease-causing gene variants that
are certain to be passed down from generation to generation.
 In GGT germ cells (sperm or egg cells) are modified by the introduction of functional genes into their
genomes. Hence, statement 1 is correct.
 Modifying a germ cell causes all the organism‘s cells to contain the modified gene.
 Germline gene therapy is not legal in many places as the risks outweigh the rewards. The change is
therefore heritable and passed on to later generations.
 It occurs in the germline cells of the human body. Generally, this method is adopted to treat the genetic,
disease-causing-variations of genes which are passed from the parents to their children. The process
involves introducing healthy DNA into the cells responsible for producing reproductive cells, eggs, or
sperms. Germline gene therapy is not legal in many places as the risks outweigh the rewards. Hence,
statement 2 is correct.
Q 68.A
 Recent Context - An exploration carried out by Geological Survey of India (GSI) has found promising
concentration of Vanadium metal in Arunachal Pradesh. Geologists are confident of identifying a deposit
soon.
 Vanadium - Vanadium in its pure form is a soft, grey and ductile element primarily derived from mined
iron ore, carbonaceous shale or phyllites and steel slag. It is a ductile transition metal with a natural
resistance to corrosion and stability against alkalis, acids and salt water.
 Vanadium has an atomic number of 23, (which means it has 23 protons in its nucleus). Its atomic symbol
is V. Natural vanadium is a combination of two isotopes, 50V (0.24 percent) and 51V (99.76 percent). It
is known as a transition metal on the periodic table.
26 www.visionias.in ©Vision IAS
 Uses:
o It is a high-value metal used in strengthening steel and titanium. Its addition improves the tensile
strength of steel and of reinforcing bars used for buildings, tunnels and bridges. Vanadium alloys are
durable in extreme temperature and environments, and are corrosion-resistant. Hence statement 1 is
correct.
o It is used in automotive and aviation industries as it increases fuel-efficiency due to its high strength-
to-weight ratio.
o It is used in vanadium redox batteries that have the least ecological impact in energy storage.
 Vanadium in India
o India is a significant consumer of vanadium but is not a primary producer of the strategic metal. It is
recovered as a by-product from the slag collected from the processing of vanadiferous magnetite ores
(iron ore)
o According to data provided by GSI, India consumed 4% of about 84,000 metric tonnes of vanadium
produced across the globe in 2017. China, which produces 57% of the world‘s vanadium, consumed
44% of the metal. China is the largest producer and consumer of this high-value metal. Hence
statement 2 is not correct.
o Vanadium mineralisation in Arunachal Pradesh is geologically similar to the ―stone coal‖ vanadium
deposits of China hosted in carbonaceous shale. This high vanadium content is associated with
graphite with fixed carbon content of up to 16%.
o The global demand for vanadium has been skyrocketing but there was a deficit of 17,300 metric
tonnes between demand and supply in 2017. The vanadium find in Arunachal Pradesh could help
boost the local and national economy
Q 69.A
 Recent context - Chief India Economist at JP Morgan has stated that the prospects of a K-shaped
recovery from COVID are increasing both in India and across the world.
 Various shapes of economic recovery: Economic recovery usually takes on in the following distinct
shapes L, U, W, V, Z and K
o L-shape - This scenario exhibits a sharp decline in the economy, followed by a slow recovery period.
It‘s often punctuated by persistent unemployment, taking several years to recoup back to previous
levels.
o U-shape - Also referred to as the ―Nike Swoosh‖ recovery, in this scenario the economy stagnates for
a few quarters and up to two years, before experiencing a relatively healthy rise back to its previous
peak.
o W-shape -This scenario offers a tempting promise of recovery, dips back into a sharp decline, and
then finally enters the full recovery period of up to two years. This is also known as a ―double-dip
recession―, similar to what was seen in the early 1980s.
o V-shape -In this best-case scenario, the sharp decline in the economy is quickly and immediately
followed by a rapid recovery back to its previous peak in less than a year, bolstered especially by
economic measures and strong consumer spending.
o Z-shape - It is defined by a boom after pent-up demand. It is considered even more optimistic than a
V-shaped recovery.
o K-shape -A K-shaped recovery happens when different sections of an economy recover at starkly
different rates. Hence option (a) is the correct answer.

Q 70.C
 Recent Context - Exquisite Dhokra products from different tribes of India were added to Tribes India e-
marketplace.
 About Dhokra Art - Dhokra is an ancient folk art tradition prevailing in India in the eastern states
of West Bengal, Odisha, Jharkhand and Chhattisgarh. While Dhokra art originated in West Bengal
(by Dhokra Damar Tribes), over time the tribes moved to Madhya Pradesh, Jharkhand, Orissa and
Chhattisgarh. Hence statement 1 is correct.
 Dhokra art is a non-ferrous metal casting style which uses the lost-wax technique. Hence statement
2 is correct.
o ‗Lost wax‘ technique is a method of metal casting in which a molten metal is poured into a mold that
has been created by means of a wax model.
o The wax figures were first covered with a coating of clay and allowed to dry.
o Then the wax was heated and the molten wax was drained out through a tiny hole made in the clay
cover.

27 www.visionias.in ©Vision IAS


o The hollow mould thus created was filled with molten metal which took the original shape of the
object. Once the metal cooled, the clay cover was completely removed.
 This style of metal casting has been in vogue in all parts of India for centuries. The oldest specimen using
such technique was during Harappan Period (famous Dancing Girl of Mohenjo-daro).
 There are several processes involved in the making of Dhokra art and hence, a single piece could take up
to a month or two to be created. Thus, it is labour-intensive process.
 Some of the examples of Dhokra metal work are - statues of fish, elephants and hangers in the boat jail
design from the Lohra tribes of Jharkhand; Ganesha statues, including a dancing Ganesha, a mask of
Goddess Durga and Lord Jagannath, a Buddha Jaali and beautiful diyas in different shapes from the
Sadeibaraini tribals from Odisha
 Tribes India e-marketplace - It is India‘s largest handicraft and organic products marketplace. It will
connect 5 lakh tribal enterprises to national and international markets, showcases tribal produce and
handicrafts, making them accessible to customers across the country.

Q 71.D
 Capillary action is defined as the spontaneous flow of a liquid into a narrow tube or porous material.
 This movement does not require the force of gravity to occur. It often acts in opposition to gravity.
 Capillary action is sometimes called capillary motion, capillarity, or wicking. Capillary action is caused
by the combination of cohesive forces of the liquid and the adhesive forces between the liquid and tube
material.
 Cohesion and adhesion are two types of intermolecular forces. These forces pull the liquid into the tube.
In order for wicking to occur, a tube needs to be sufficiently small in diameter.
 Examples of capillary action include:
 Capillary action in plants is one of the most essential processes which helps them to survive. The roots of
plants draw water via capillary action from the soil, and thus, supply it to various parts of the plant
including the stems, branches, and leaves. Hence option 3 is correct.
 A candle burns due to capillary action. When the wick of a candle is lit, the heat of the flame causes the
candle wax to melt. The molten wax is then pulled up through the wick by capillary action and supplies
the fuel needed to sustain the fire and keep the candle burning. Hence option 2 is correct.
 Pores in the sponge act as minute capillaries, causing them to absorb a large number of liquids. Hence
option 1 is correct.
 Other examples include uptake of water in paper and plaster (two porous materials), the movement of
water through sand etc.
Q 72.A
 Recent Context - Researchers at the Indian Institute of Science Education and Research (IISER) have
identified a gene – ‗BBX11‘ that facilitates in the greening of plants.
 About ‗BBX11‘
o The synthesis of chlorophyll in plants is a lengthy, multi-step process. When a seedling emerges from
under the soil it must quickly synthesize chlorophyll to start supporting its own growth.
o In order to facilitate the quick synthesis of chlorophyll, plants make a precursor of chlorophyll called
‗protochlorophyllide‘ in the dark, which glows red when blue light is shone on the plant.
o As soon as the plant comes out into the light from under the soil, light-dependent enzymes convert
protochlorophyllide to chlorophyll.
o Researchers found that the two proteins oppositely regulate the ‗BBX11‘ gene to maintain optimum
levels of ‗BBX11‘. Thus, it facilitates in the greening of plants. Hence option (a) is the correct
answer.
o It plays a crucial role in regulating the levels of protochlorophyllide — an intermediate in the
biosynthesis of the green pigment chlorophyll.
o The amount of protochlorophyllide synthesized needed to be proportional to the number of enzymes
available to convert them to chlorophyll. If there is an excess of free protochlorophyllide, then
exposure to light converts it into molecules that cause ‗photobleaching‘.
o Thus, it is very important to regulate the amount of protochlorophyllide synthesized by the plant and
here comes the vital plant played by the ‗BBX11‘ gene. If it is less, plants are unable to efficiently
‗green‘ in order to harvest sunlight.
 Significance
o The study could have tremendous implications in the agriculture sector in tropical countries like India
and can help provide leads to optimize plant growth under stressful and rapidly changing climatic
conditions.
28 www.visionias.in ©Vision IAS
Q 73.D
 Benzo[a]pyrene is a polycyclic aromatic hydrocarbon and the result of incomplete combustion of
organic matter at temperatures between 300 °C (572 °F) and 600 °C (1,112 °F).
 The main source of atmospheric BaP is residential wood burning. It is also found in coal tar, in
automobile exhaust fumes (especially from diesel engines). BaP is discharged in wastewater by industries
such as smelters, particularly iron and steel mills and aluminum smelters, in all smoke resulting from the
combustion of organic material (including cigarette smoke), and in charbroiled food or grilled meats.
 A 2001 National Cancer Institute study found levels of BaP to be significantly higher in foods that were
cooked well-done on the barbecue, particularly steaks, chicken with skin, and hamburgers. BaP has been
identified as a prime carcinogen in cigarette smoke
 According to the National Ambient Air Quality Standards (NAAQS), there are twelve major pollutants
including PM 10, PM 2.5, Nitrogen Dioxide (NO2), Sulphur Dioxide (SO2), Carbon Monoxide (CO),
Ozone (O3), Ammonia (NH3), Benzene (C6H6), Benzo (a) Pyrene (BaP), Arsenic (As), Nickel (Ni) and
Lead (Pb). Hence the correct option is (d)

Q 74.C
 Biominerals are natural composite materials based upon biomolecules (such as proteins) and minerals
produced by living organisms via processes known as biomineralization.
 Biominerals can be formed from a number of metal ions, including Ca, Fe, Mg and Mn with appropriate
anions such as carbonate, phosphate, oxalate, sulphate, oxides and sulphides. More than 60 different
biological minerals are known. Calcium carbonates and phosphates alone constitute a major part of the
biomineral world, including bones and teeth
 Biomineralization: Complete conversion of organic substances to inorganic derivatives by living
organisms, especially micro-organisms, yielding materials with impressive mechanical properties
such as bones, shells and teeth. Hence the correct option is (c)

Q 75.A
 Gravitational waves are 'ripples' in space-time caused by some of the most violent and energetic processes
in the Universe. They are created when:
o The strongest gravitational waves are produced by cataclysmic events such as colliding black holes,
supernovae (massive stars exploding at the end of their lifetimes), and colliding neutron stars.
o Other waves are predicted to be caused by the rotation of neutron stars that are not perfect spheres.
o Some are possibly created even by the Big Bang and are the remnants of gravitational radiation
created then. So not all Gravitational waves are as old as Big Bang. Hence statement 1 is not
correct.
 They travel at the speed of light, gravitational waves squeeze and stretch anything in their path. Hence
statement 2 is not correct.
 Gravitational waves were proposed by Albert Einstein in his General Theory of Relativity over a
century ago. It was only in 2015, however, that the first gravitational wave was actually detected — by
LIGO. Since then, there have been a number of subsequent detections of gravitational waves. Hence
statement 3 is correct.

Q 76.C
 In 2018, the definitions of the kilogram, the ampere, the kelvin and the mole were changed.
 The kilogram‘s definition will be based on a concept of physics called the Planck constant. The Planck
constant is a quantity that relates a light particle‘s energy to its frequency.
 The new measure for kilogram from 2019 derived from constants of nature that are all interdependent. ‎It
will be set by the value of Planck constant in combination with definitions of meter and second. Kibble
balance will be used to make accurate measures of Planck constant. ‎Kibble balance is a set of scales,
which uses force produced by current-carrying wire in a magnetic field to balance the weight of the mass.
 Apart from Mass, there are six base quantities as per the SI (International System of Units).
 The International System of Units (SI), commonly known as the metric system, is the international
standard for measurement. The International Treaty of the Meter was signed in Paris on May 20, 1875, by
seventeen countries, including the United States and is now celebrated around the globe as World
Metrology Day.
 The SI rests on a foundation of seven (7) defining constants: the caesium hyperfine splitting frequency,
the speed of light in vacuum, the Planck constant, the elementary charge (i.e. the charge on a proton), the
Boltzmann constant, the Avogadro constant, and the luminous efficacy of a specified monochromatic
29 www.visionias.in ©Vision IAS
source. Definitions of all seven (7) SI base units are expressed using an explicit-constant formulation and
experimentally realized using a specific mises en pratique (practical technique).
 The seven SI base units, which are comprised of:
o Length - meter (m)
o Time - second (s)
o Amount of substance - mole (mole)
o Electric current - ampere (A)
o Temperature - kelvin (K)
o Luminous intensity - candela (cd)
o Mass - kilogram (kg)
 Hence only options 1, 2 and 4 are correct.

Q 77.A
 Mineral nutrients are essential for the growth of the plants. The absence of any mineral nutrient leads to
abnormalities in the growth of the plant.
 The functions of some of the important macro (required in large amounts) and micro (required in small
amounts) mineral nutrients are as follows-
 Magnesium is a critical structural component of the chlorophyll molecule and is necessary for
functioning of plant enzymes to produce carbohydrates, sugars and fats. It is used for fruit and nut
formation and essential for germination of seeds. Deficient plants appear chlorotic, show yellowing
between veins of older leaves; leaves may droop. Magnesium is leached by watering and must be
supplied when feeding. It can be applied as a foliar spray to correct deficiencies. Hence the correct
option is (a)
 Sulfur is a structural component of amino acids, proteins, vitamins and enzymes and is essential to
produce chlorophyll. It imparts flavor to many vegetables. Deficiencies show as light green leaves. Sulfur
is readily lost by leaching from soils and should be applied with a nutrient formula. Some water supplies
may contain Sulfur.
 Calcium activates enzymes, is a structural component of cell walls, influences water movement in cells
and is necessary for cell growth and division. Some plants must have calcium to take up nitrogen and
other minerals. Calcium is easily leached. Calcium, once deposited in plant tissue, is immobile (non-
translocatable) so there must be a constant supply for growth. Deficiency causes stunting of new growth
in stems, flowers and roots. Symptoms range from distorted new growth to black spots on leaves and fruit.
Yellow leaf margins may also appear.
 Iron is necessary for many enzyme functions and as a catalyst for the synthesis of chlorophyll. It is
essential for the young growing parts of plants. Deficiencies are pale leaf color of young leaves followed
by yellowing of leaves and large veins. Iron is lost by leaching and is held in the lower portions of the soil
structure. Under conditions of high pH (alkaline) iron is rendered unavailable to plants. When soils are
alkaline, iron may be abundant but unavailable. Applications of an acid nutrient formula containing iron
chelates, held in soluble form, should correct the problem.
 The various life processes of living organisms require at least 40 elements in different quantities.
Elements that are needed in large amount are called macronutrients. For example, carbon, hydrogen,
oxygen (components of all organic compounds), nitrogen, potassium, calcium, sodium, magnesium, iron,
phosphorous and sulphur. They all perform various functions.
 Substances required in small amounts are called micronutrients. They include copper, manganese,
selenium, zinc, molybdenum, boron, silicon, etc. Some of the micronutrients (e.g. Zinc, Boron) may often
serve as limiting factors to affect the growth, survival, and propagation of living organisms

Q 78.C
 Thorium is more abundant in nature than uranium. It is fertile rather than fissile. This means
Thorium cannot be converted into fissile material by irradiation in a reactor. It can only be used as a fuel
in conjunction with a fissile material such as recycled plutonium. Hence statement 1 is correct.
 Thorium fuels can breed fissile uranium-233 to be used in various kinds of nuclear reactors.
 It is estimated that one ton of thorium can produce as much energy as 35 tons of uranium in a liquid
fluoride thorium reactor. Conventional reactors utilizes less than one percent of uranium, whereas a well
working reprocessing reactor can utilize 99% of its thorium fuel. Hence statement 2 is correct.
 Thorium-based reactors are safer because the reaction can easily be stopped and because the operation
does not have to take place under extreme pressures. Compared to uranium reactors, thorium reactors
produce far less waste and the waste that is generated is much less radioactive and much shorter-lived.
30 www.visionias.in ©Vision IAS
Q 79.D
 Jiuzhang is a new quantum computer which is developed by the Chinese research team in
December, 2020, Tangle Lake is Intel's new superconducting quantum chip and Sycamore is
quantum computer which was developed by Google in 2019. These discoveries were based on
quantum technology. Hence option (d) is the correct answer.
 Quantum Technology: It is a class of technology that works by using the principles of quantum mechanics
(the physics of sub-atomic particles), which revolves around wave-particle duality of matter. Ny using
quantum technology, scientists are developing Quantum Computing, as an ordinary computer chip uses
bits which hold information in the form of 0 (off) or 1 (on). Instead of bits, quantum computers use
quantum bits or qubits.
 This helps it in following ways:
o Superposition (qubit can take values of 0 or 1 or range from 0 to 1): It is the feature of a quantum
system whereby it exists in several separate quantum states at the same time . unlike normal computer
which rules out wrong methods to find the right one, a quantum computer can go down every method
at once. So, they even carry potential to crack today‘s encryption easily.
o Entanglement: It is a quantum mechanical phenomenon in which the quantum states of two or more
objects have to be described with reference to each other, even though the individual objects may be
spatially separated. In such way, information at one side of wave function can be used to interpret at
the other, even if they are physically separate.

Q 80.B
 World Malaria Report (WMR) 2020 released by World Health Organisation (WHO), which gives the
estimated cases for malaria across the world.
 The report highlighted that India achieved a reduction of 83.34% in malaria morbidity and 92% in malaria
mortality between the year 2000 and 2019. Report recognised the effort of WHO‘s initiative ―High burden
to high impact‖. ―High burden to high impact‖ is a country-led approach that is supported by WHO, the
RBM Partnership to End Malaria in 11 high malaria burden countries, including India. The
Implementation of ―High Burden to High Impact (HBHI)‖ initiative has been started in four Indian
states i.e., West Bengal, Jharkhand, Chhattisgarh and Madhya Pradesh in July, 2019. Hence option
(b) is the correct answer.
 About Malaria: Malaria is caused by Plasmodium parasites. The parasites are spread to people through the
bites of infected female Anopheles mosquitoes, called "malaria vectors." There are 5 parasite species that
cause malaria in humans, and 2 of these species – P. falciparum and P. vivax – pose the greatest threat.

Q 81.A
 An adiabatic process is a process in which there is no transfer of heat between the system and
surroundings. Here, the wall separating the system and the surroundings is called the adiabatic wall. The
adiabatic process can be either reversible or irreversible.
 The adiabatic lapse rate is the rate at which the temperature of an air parcel changes in response to the
compression or expansion associated with elevation change, under the assumption that the process is
adiabatic, i.e., no heat exchange occurs between the given air parcel and its surroundings.
 Adiabatic lapse rate: Change of temperature with a change in altitude of an air parcel without
gaining or losing any heat to the environment surrounding the parcel.
 Adiabatic processes are characterized by an increase in entropy, or degree of disorder, if they are
irreversible and by no change in entropy if they are reversible. Adiabatic processes cannot decrease
entropy.

Q 82.C
 Intermolecular forces are the forces of attraction and repulsion between interacting particles (atoms
and molecules). This term does not include the electrostatic forces that exist between the two oppositely
charged ions and the forces that hold atoms of a molecule together i.e., covalent bonds.
 Attractive intermolecular forces are known as van der Waals forces, in honour of Dutch scientist
Johannes van der Waals (1837- 1923), who explained the deviation of real gases from the ideal behaviour
through these forces. Hence the correct option is (c)
 Van der Waals forces vary considerably in magnitude and include dispersion forces or London forces,
dipole-dipole forces, and dipole-induced dipole forces. A particularly strong type of dipole-dipole
interaction is hydrogen bonding.

31 www.visionias.in ©Vision IAS


 The electrostatic force is also known as the Coulomb force or Coulomb interaction. It's the attractive
or repulsive force between two electrically charged objects.
 In nuclear physics and particle physics, the weak interaction, which is also often called the weak force
or weak nuclear force, is the mechanism of interaction between subatomic particles that is responsible
for the radioactive decay of atoms.
 Newton‘s Law of Universal Gravitation is used to explain the gravitational force. This law states that
every massive particle in the universe attracts every other massive particle with a force which is directly
proportional to the product of their masses and inversely proportional to the square of the distance
between them.

Q 83.C
 The octet rule states that the element will gain or lose electrons in order to have a full other shell of eight
electrons. Most elements strive to be stable like their nearest Noble gases. To do this they either lose or
gain electrons. This is done by bonding either ionically, metallically or covalently.

Q 84.B
 Graphene is a two-dimensional carbon allotrope. It is composed of carbon atoms positioned in a
hexagonal design, which can be said to resemble a chicken wire. Graphene is the basic building block for
other graphitic materials; it also represents a conceptually new class of materials that are only one atom
thick, so-called two-dimensional (2D) materials (they are called 2D because they extend in only two
dimensions: length and width; as the material is only one atom thick Hence statement 1 is not correct
and statement 2 is correct.
 It is the building block for other graphitic materials (since a typical carbon atom has a diameter of about
0.33 nanometers, there are about 3 million layers of graphene in 1 mm of graphite).
 Units of graphene are known as nanographene; these are tailored to specific functions and as such their
fabrication process is more complicated than that of generic graphene. Nanographene is made by
selectively removing hydrogen atoms from organic molecules of carbon and hydrogen, a process called
dehydrogenation.
 Harder than diamond yet more elastic than rubber; tougher than steel yet lighter than aluminium.
Graphene is the strongest known material.
 Graphene possesses other amazing characteristics: Its high electron mobility is 100x faster than silicon; it
conducts heat 2x better than diamond; its electrical conductivity is 13x better than copper; it absorbs only
2.3% of reflecting light. One of the most useful properties of graphene is that it is a zero-overlap
semimetal (with both holes and electrons as charge carriers) with very high electrical conductivity.
Hence statement 3 is correct.
 Graphene uses and applications
o Energy storage and solar cells
o Sensor applications
o Graphene ink
o Transistors and memory
o Flexible, stretchable and foldable electronics
o Photodetectors
o Coatings

Q 85.D
 With the rapid rise in cases of COVID-19, Indian government approved multiple tests for faster detection
and wider coverage of infected person. The tests used for detecting COVID-19 are:
 Reverse transcription polymerase chain reaction (RT-PCR) test: The real-time polymerase chain
reaction (RT-PCR) test is the most commonly used one for Covid-19. The RT-PCR test starts with a
sample swab taken from inside a person‘s throat or nose. Coronaviruses have RNA or ribonucleic acid as
their genetic material.
o Researchers select specific areas in the genome that do not mutate rapidly as the virus evolves and
create copies of these using the PCR process. Reverse transcription polymerase chain reaction (RT-
PCR) is a laboratory technique combining reverse transcription of RNA into DNA (called
complementary DNA or cDNA) and amplification of specific DNA targets using polymerase chain
reaction (PCR).
o It is the most sensitive technique for mRNA detection. It quantifies the presence of a particular
genetic material of the virus or any pathogen. (Viruses like SARS-CoV2 with genomes stored as RNA

32 www.visionias.in ©Vision IAS


are collected by swab from throat or inside the nose. Then the RNA — a single-strand molecule — is
converted into a two-strand DNA using an enzyme. This is known as reverse transcription. Hence
option 2 is correct.
 Antigen test: These tests are designed to detect a specific protein in the virus that elicits the body‘s
immune response. In the case of Covid-19, it is the ‗spike protein‘ present on the surface of the
coronavirus that facilitates its entry into the human cell. For this test, Health exert collect a nasal swab,
which is then immersed in a solution that deactivates the virus. Hence option 1 is correct.
 Antibody tests, also known as serological tests, detect whether a person has antibodies to the virus.
Antibodies are naturally produced by the body‘s immune system to fight off infections. Two specific
antibody types are sought out in an antibody test:
o IgM antibodies against SARS-CoV-2, which develops early on in an infection.
o IgG antibodies against SARS-CoV-2, which are mostly found after someone has recovered from the
infection. On the basis of these antibodiesm results are concluded. However, Antibody tests cannot be
used to diagnose Covid-19 but it can reveal whether a person was recently exposed to the
virus. Hence option 4 is correct.
 TrueNAT: ICMR allowed the use of the TrueNAT system for screening and confirmation of Covid-19.
The chip-based test works by detecting the SARS-CoV-2 E-gene, which helps build the envelope that
holds the components of the coronavirus together. TrueNAT is an indigenously developed, portable
version of CB-NAAT or Cartridge Based Nucleic Acid Amplification Test, also known as the Genexpert
test. Both these tests were initially developed for testing tuberculosis and currently are being used for
Covid-19 test too. Hence options 3 and 5 are correct.

Q 86.C
 Electromagnetic radiation is a type of energy that is commonly known as light. The light travels in
waves, and all electromagnetic radiation travels at the same speed which is about 3.0 * 108 meters per
second through a vacuum. It is commonly known as the "speed of light"; nothing can move faster than
the speed of light. Hence statement 3 is correct.
 A wavelength is one cycle of a wave. It is measured as the distance between any two consecutive
peaks (is the highest point of the wave) of a wave. If the wavelength of a light wave is shorter, that
means that the frequency will be higher because one cycle can pass in a shorter amount of time.
 Light waves with shorter wavelengths (and higher frequencies) have more energy, so types of light
like gamma rays, X-rays, and ultraviolet light are more energetic than visible light, and infrared,
microwaves, and radio waves are less energetic than the light in the visible spectrum. Hence statement 1
is correct and 2 is not correct.
Q 87.C
 Specialized roots:
o The roots, or parts of roots, of many plant species, have become specialized to serve adaptive
purposes and some of the specialized roots are as follows:
 Adventitious roots arise from the stem, branches, leaves, or old woody roots. E.g., willow.
 Pneumatophores (Aerating roots) are the roots raised above the ground, especially above water.
Pneumatophores are the roots that facilitate a gas exchange under anaerobic soil conditions. E.g.,
Avicennia. Hence statement 1 is correct.
 Aerial roots are the roots found entirely above the ground. E.g., Epiphytic orchids.
 Haustorial roots are the roots of parasitic plants that can absorb water and nutrients from another
plant. E.g., Dodder. Hence statement 2 is correct.
Q 88.B
 Allotropy or allotropism is the property of some chemical elements to exist in two or more different
forms, in the same physical state, known as allotropes of the elements. Hence statement 1 is not
correct.
 Allotropes are different structural modifications of an element; the atoms of the element are bonded
together in a different manner. For example, the allotropes of carbon include diamond (the carbon
atoms are bonded together in a tetrahedral lattice arrangement), graphite (the carbon atoms are bonded
together in sheets of a hexagonal lattice), graphene (single sheets of graphite), and fullerenes (the carbon
atoms are bonded together in spherical, tubular, or ellipsoidal formations). Hence statement 2 is correct.
 The term allotropy is used for elements only, not for compounds. The more general term, used for any
crystalline material, is polymorphism. Allotropy refers only to different forms of an element within the
same phase (i.e.: solid, liquid, or gas states); the differences between these states would not alone
constitute examples of allotropy.

33 www.visionias.in ©Vision IAS


Q 89.C
 Plaster of Paris, quick-setting gypsum plaster consisting of a fine white powder (calcium sulfate
hemihydrate), which hardens when moistened and allowed to dry. Known since ancient times, plaster of
paris is so called because of its preparation from the abundant gypsum found near Paris.
 Plaster of paris is prepared by heating (de-hydration) calcium sulfate dihydrate, or gypsum, to 120–
180 °C (248–356 °F) with an additive to retard the set, it is called wall, or hard wall, plaster, which
can provide passive fire protection for interior surfaces.
 Gypsum can also help improve soil structure. Adding gypsum to the soil reduces erosion by increasing the
ability of soil to soak up water after precipitation, thus reducing runoff.
 Gypsum application also improves soil aeration and water percolation through the soil profile. A
recent study showed the benefit of gypsum application to improve movement of water through the profile
to tile drains. It also reduces phosphorus movement out of the field. Hence the correct option is (c)

Q 90.C
 A transverse wave is a wave that vibrates perpendicular to the direction of the wave or path of
propagation. A simple example is given by the waves that can be created on a horizontal length of the
string by anchoring one end and moving the other end up and down.
 In a transverse wave, the particles are displaced perpendicular to the direction the wave travels. In a
longitudinal wave, the particles are displaced parallel to the direction the wave travels. An example of
longitudinal waves is compressions moving along a slinky. Hence statement 1 is not correct.
 Mechanical transverse waves cannot propagate in a gaseous or a liquid medium because there is no
mechanism for driving motion perpendicular to the propagation of the wave. Hence statement 2 is
correct.
 On the other hand, the common electromagnetic waves (unlike mechanical waves), like visible light do
not require a medium to travel and thus can travel in gaseous medium as well as in vacuum.
 Transverse waves commonly occur in elastic solids; the oscillations, in this case, are the displacement of
the solid particles away from their relaxed position, in directions perpendicular to the propagation of the
wave. Since those displacements correspond to local shear deformation of the material, a transverse wave
of this nature is called a shear wave. In seismology, shear waves are also called secondary waves or S-
waves. Hence statement 3 is correct.

Q 91.B
 Lipid Digestion in the Small Intestine: As the stomach contents enter the small intestine, most of the
dietary lipids are undigested and clustered in large droplets. Bile, which is made in the liver and stored in
the gallbladder, is released into the duodenum, the first section of the small intestine. Bile salts have both
a hydrophobic and a hydrophilic side, so they are attracted to both fats and water. This makes them
effective emulsifiers, meaning that they break large fat globules into smaller droplets. Emulsification
makes lipids more accessible to digestive enzymes by increasing the surface area for them to act.
 The pancreas secretes pancreatic lipases into the small intestine to enzymatically digest triglycerides.
Triglycerides are broken down into fatty acids, monoglycerides (glycerol backbone with one fatty acid
still attached), and some free glycerol. Cholesterol and fat-soluble vitamins do not need to be
enzymatically digested.
 Most of the lipids in our food are present in the form of simple fats or triglycerides.
 Lipase enzyme plays an important role in the digestion of lipids. These breakdown lipids into fatty
acids and monoglycerides.
 Bile probably contains no digestive enzyme, yet it plays an important role in the digestion and absorption
of fat. It facilitates the emulsification of fats.
 Pepsin is a protein-digesting or proteolytic enzyme. Hence option (b) is the correct answer.

Q 92.D
 What is a zoonotic disease?
o A zoonotic disease is a disease spread between animals and people. Zoonotic diseases can be caused
by viruses, bacteria, parasites, and fungi. Some of these diseases are very common.
 Some examples include:
o Anthrax, Ascariasis, Brucellosis, Plague, Echinococcosis, Lassa fever, Listeriosis, Lyme disease,
Monkeypox, Psittacosis, Rabies, Salmonellosis, Trichinosis, Toxoplasmosis, Typhus, West Nile
fever. Hence option (d) is the correct answer.
 Major modern diseases such as Ebola virus disease and salmonellosis are zoonoses.
34 www.visionias.in ©Vision IAS
 HIV was a zoonotic disease transmitted to humans in the early part of the 20th century, though it has now
evolved to a separate human-only disease.
 Anthrax: Anthrax is primarily a disease of herbivorous mammals, although other mammals and some
birds have been known to contract it. Until the introduction and widespread use of effective veterinary
vaccines, it was a major cause of fatal disease in cattle, sheep, goats, camels, horses, and pigs throughout
the world.
 Humans generally acquire the disease directly or indirectly from infected animals, or occupational
exposure to infected or contaminated animal products. Control in livestock is therefore the key to reduced
incidence. The disease is generally regarded as being non-contagious. Records of person-to-person spread
exist, but are rare.
 Plague: Plague is an infectious disease caused by Yersinia pestis bacteria, usually found in small
mammals and their fleas. The disease is transmitted between animals via their fleas and, as it is a zoonotic
bacterium, it can also transmit from animals to humans.
o Humans can be contaminated by the bite of infected fleas, through direct contact with infected
materials, or by inhalation. Plague can be a very severe disease in people, particularly in its
septicaemic and pneumonic forms, with a case-fatality ratio of 30% - 100% if left untreated.
 Brucellosis: Brucellosis is a bacterial disease caused by various Brucella species, which mainly infect
cattle, swine, goats, sheep and dogs. Humans generally acquire the disease through direct contact with
infected animals, by eating or drinking contaminated animal products, or by inhaling airborne agents.
The majority of cases are caused by ingesting unpasteurized milk or cheese from infected goats or
sheep. Person-to-person transmission is rare. The disease causes flu-like symptoms, including fever,
weakness, malaise and weight loss.
 Salmonellosis:
o Salmonellosis is a disease caused by the bacteria Salmonella. It is usually characterized by acute
onset of fever, abdominal pain, diarrhoea, nausea and sometimes vomiting.
o Symptoms of salmonellosis are relatively mild and patients will make a recovery without specific
treatment in most cases. However, in some cases, particularly in children and elderly patients, the
associated dehydration can become severe and life-threatening.
o Sources and transmission Salmonella bacteria are widely distributed in domestic and wild
animals. They are prevalent in food animals such as poultry, pigs, and cattle; and in pets,
including cats, dogs, birds, and reptiles such as turtles.
 West Nile Virus (WNV):
o West Nile Virus can cause neurological disease and death in people. WNV is maintained in nature in
a cycle involving transmission between birds and mosquitoes. Humans, horses and other mammals
can be infected. West Nile Virus (WNV) is a member of the flavivirus genus and belongs to the
Japanese encephalitis antigenic complex of the family Flaviviridae.
o Transmission Human infection is most often the result of bites from infected mosquitoes.
Mosquitoes become infected when they feed on infected birds, which circulate the virus in their blood
for a few days. The virus eventually gets into the mosquito's salivary glands. During later blood meals
(when mosquitoes bite), the virus may be injected into humans and animals, where it can multiply and
possibly cause illness.
o The virus may also be transmitted through contact with other infected animals, their blood, or other
tissues.

Q 93.A
 The International Union of Pure and Applied Chemistry (IUPAC) is an international federation of
National Adhering Organizations that represents chemists in individual countries. It is a member of the
International Science Council (ISC). IUPAC is registered in Zürich, Switzerland, and the administrative
office, known as the "IUPAC Secretariat".
 IUPAC was established in 1919 as the successor of the International Congress of Applied Chemistry for
the advancement of chemistry. The IUPAC has the responsibility of naming all new elements and
compounds, declare atomic weights and physical constants, update the Periodic table and carry out
research projects for the benefit of mankind. Hence statement 1 is correct and statement 2 is not
correct.
 IUPAC has publications in many scientific fields including chemistry, biology and physics. Some
important work IUPAC has done in these fields includes standardizing nucleotide base sequence code
names; publishing books for environmental scientists, chemists, and physicists; and improving education
in science. IUPAC is also known for standardizing the atomic weights of the elements through one of its
oldest standing committees, the Commission on Isotopic Abundances and Atomic Weights (CIAAW).
35 www.visionias.in ©Vision IAS
 Recently, Dr. Bipul Behari Saha was elected as the Bureau Member of the International Union of
Pure and Applied Chemistry (IUPAC) for 2020-23. In the 100 years history of the union, this is the
second time an Indian scientist is elected as a Bureau Member, after Bharat Ratna Prof. CNR Rao in
1979.

Q 94.C
 Radio waves have the longest wavelengths in the electromagnetic spectrum. They range from the
length of a football to larger than our planet. have frequencies as high as 300 gigahertz to as low as 30
hertz. At 300 GHz, the corresponding wavelength is 1 mm; at 30 Hz the corresponding wavelength is
10,000 km. Hence statement 1 is correct.
 WiFi works off of the same principal as other wireless devices - it uses radio frequencies to send signals
between devices. The radio frequencies are completely different say from walky-talkies, car radios, cell
phones, and weather radios. For WiFi, this frequency happens to be 2.4Ghz and 5Ghz. Hence statement 2
is correct.
 Radio waves with wavelengths of kilometres diffract over hills and through valleys so can easily get
reception.
 But microwaves with a wavelength of centimetres tend to go straight. Hence statement 3 is not
correct.

Q 95.D
 The Korea Superconducting Tokamak Advanced Research (KSTAR), a superconducting fusion
device also known as the Korean artificial sun, set the new world record as it succeeded in
maintaining the high-temperature plasma for 20 seconds with an ion temperature over 100 million
degrees (Celsius).
 For the fusion, scientists obtained a plasma (one of the four fundamental states of matter) from
hydrogen -- composed of hot ions that exceeded the 100 million degrees temperature. To retain the
ions, it is necessary to maintain incredibly high temperatures.
 The researchers established a world record by turning this national artificial sun nuclear fusion reactor on
and maintaining the high-temperature plasma for 20 seconds with an ion temperature over 100 million
degrees (Celsius).

Q 96.B
 Recent Context – Recently, Climate Change Performance Index (CCPI) – 2020 was released by non –
profit Germanwatch.
 Climate Change Performance Index (CCPI) - It is an independent monitoring tool for tracking
countries‘ climate protection performance. The Index is published by Germanwatch, the New Climate
Institute and the Climate Action Network. It is being published annually since 2005. Hence statement 1
is not correct.
 It indicates the climate mitigation efforts of 57 countries and the EU, which collectively account for over
90% of global greenhouse gas (GHG) emissions. Results are compiled with the aid of 400 national experts
evaluating their countries‘ most recent national and international climate policies.
 It assesses countries‘ performance in four categories:
o GHG Emission (40% of overall score)
o Renewable Energy (20% of overall score)
o Energy Use (20% of overall score)
o Climate Policy (20% of overall score)
 Hence statement 2 is correct.
 The CCPI provides long-standing and reliable analysis conducted on the leaders and laggards in climate
protection. This makes it usable as a powerful instrument to hold governments accountable for their
responsibility to act on the climate crisis, to inform the process of raising climate ambition, and to
stimulate a race to the top in climate action.
 It helps to access and judge countries‘ climate policy, their recent development, current levels and well-
below-2 degrees celsius compatibility of greenhouse gas (GHG) emissions, renewable energies, energy
use and their targets for 2030.
 CCPI – 2020
 As per the index, no country was doing enough to meet the goals of the 2015 Paris Agreement. The USA
was the worst performer. Six G20 countries were ranked among very low performers.

36 www.visionias.in ©Vision IAS


 India has been placed at the tenth position in the index (score – 63.98/100). In 2019, India has been ranked
at the ninth position, with an overall score of 66.02. thus, for the second time in a row, India continued to
remain in the top 10 ranks. Hence statement 3 is correct.
 India received high ratings on all CCPI indicators except ‗renewable energy‘, where it was categorised as
having a ‗medium‘ performance.

Q 97.D
 Reactions in which heat is released along with the formation of products are called exothermic chemical
reactions. Exothermic reactions feel warm or hot or may even be explosive. More energy is released
making chemical bonds than is used to break them.
 Examples of Exothermic Reactions:
o any combustion reaction
o a neutralization reaction
o rusting of iron (rust steel wool with vinegar)
o the reaction between water and calcium chloride
o reaction between sodium and chlorine to make sodium chloride (table salt)
o reaction between water and any strong acid
o reaction between water and any anhydrous salt
o dissolving laundry detergent in water
o freezing water into ice cubes
o snow forming inside clouds
o respiration, decomposition of organic matter, photosynthesis. Hence the correct answer is
option (d).
o rain forming from water vapor in clouds
o burning sugar
o a burning candle
o nuclear fission

Q 98.B
 Option 1 is correct: Plastids are present only in plant cells. There are various types of plastids –
chromoplasts (coloured plastids) and leucoplasts (white or colourless plastids). Plastids containing the
pigment chlorophyll are known as chloroplasts.
 Option 2 is not correct: Vacuoles are storage sacs for solid or liquid contents. Vacuoles are small sized
in animal cells while plant cells have very large vacuoles. The central vacuole of some plant cells may
occupy 50-90% of the cell volume.
 Option 3 is not correct: Mitochondria present in both animal and plant cells are known as the
powerhouses of the cell. The energy required for various chemical activities needed for life is released by
mitochondria in the form of ATP (Adenosine triphopshate) molecules.

Q 99.D
 The photoelectric effect is the emission of electrons when electromagnetic radiation, such as light,
hits a material. Electrons emitted in this manner are called photoelectrons.
 The phenomenon is studied in condensed matter physics, and solid-state and quantum chemistry to draw
inferences about the properties of atoms, molecules and solids. The effect has found use in electronic
devices specialized for light detection and precisely timed electron emission.
 Devices based on the photoelectric effect have several desirable properties, including producing a current
that is directly proportional to light intensity and a very fast response time.
 One basic device is the photoelectric cell or photodiode, which can detect light, measure its intensity,
control other devices as a function of illumination, and turn light into electrical energy. These devices
work at low voltages, comparable to their bandgaps, and they are used in industrial process control,
pollution monitoring, light detection within fiber optics telecommunications networks, solar cells,
imaging, and many other applications.
 Pollution monitoring: The photoelectric aerosol sensor array as a tool for on line and in situ analysis of
heavy metal aerosols. A photoelectric aerosol sensor (PAS), an array operating with different wavelengths
is used. The photoelectric activity at each wavelength and the linearity of the PAS signal is investigated
for monodisperse and quasi-polydisperse systems. The aerosol mass concentration is determined by
correlation with atomic absorption spectroscopy measurements of filter samples. Pattern recognition is

37 www.visionias.in ©Vision IAS


the correct answer is option (d).
 The Nobel Prize in Physics 1921 was awarded to Albert Einstein "for his services to Theoretical Physics,
and especially for his discovery of the law of the photoelectric effect."

Q 100.A
 Recent Context – Recently, Ministry of Finance and UN-based Better Than Cash Alliance (BTCA)
organized a joint Peer learning exchange on fintech solutions for responsible digital payments at the last
mile.
 Better Than Cash Alliance (BTCA) - It is a partnership of over 75 governments, companies and
international organizations committed to accelerate the transition from cash to digital payments to help
achieve the Sustainable Development Goals. They are committed to digitizing payments in order to boost
efficiency, transparency, women‘s economic participation and financial inclusion, helping build
economies that are digital and inclusive. Hence statement 1 is correct.
 BTCA is funded by the Bill & Melinda Gates Foundation, Citi, Ford Foundation, MasterCard, Omidyar
Network, U.S. Agency for International Development and Visa Inc.
 United Nations Capital Development Fund (UNCDF) serves as secretariat for the BTCA.
 The Alliance Secretariat works with members on their journey to digitize payments by:
o Providing advisory services based on their priorities.
o Sharing action-oriented research and fostering peer learning on responsible practices.
o Conducting advocacy at national, regional and global level.
 India became a member of the Better Than Cash Alliance in 2015 to digitize payments and to share
success stories from Pradhan Mantri Jan Dhan Yojana, the world's largest financial inclusion program.
Hence statement 2 is not correct.

Copyright © by Vision IAS


All rights are reserved. No part of this document may be reproduced, stored in a retrieval system or transmitted
in any form or by any means, electronic, mechanical, photocopying, recording or otherwise, without prior
permission of Vision IAS.

38 www.visionias.in ©Vision IAS

You might also like